Anteckningar i Matematisk analys i en dimension, vt 2015

1
Anteckningar i Matematisk analys i en dimension, vt 2015
y
y= f HxL
n
b
1
â f HkL»à f HxLâx
b - a k=1
a
a
b
x
2
February 9, 2015
Innehåll
1 Summa
1.1 Summasymbolen med exempel . . . . . . . . . . . . . . . . . . .
1.1.1 Summasymbolen . . . . . . . . . . . . . . . . . . . . . .
1.1.2 Tre summor . . . . . . . . . . . . . . . . . . . . . . . . .
2 Integral
2.1 Integralens definition och enkla exempel . . . . . . . .
2.1.1 Integral m.h.a. summor . . . . . . . . . . . . .
2.1.2 Integralkalkylens medelvärdessats I . . . . . .
2.1.3 Integralkalkylens huvudsats (5.5) . . . . . . .
2.1.4 Integral tolkad som area . . . . . . . . . . . .
2.2 Primitiv funktion till olika funktionsklasser . . . . . .
2.2.1 Primitiv funktion till/integral av potensfunktion
2.2.2 Integral av exponentialfunktion . . . . . . . .
2.2.3 Integral av trigonometriska funktioner . . . . .
2.2.4 Linearitetsegenskaper hos integralen . . . . . .
2.3 Derivata och integral . . . . . . . . . . . . . . . . . .
2.4 Integraler forts . . . . . . . . . . . . . . . . . . . . .
2.4.1 Variabelsubstitution . . . . . . . . . . . . . .
2.4.2 V.S. t = tan(x/2) . . . . . . . . . . . . . . . .
2.5 Mer om integraler . . . . . . . . . . . . . . . . . . . .
2.5.1 Area mellan funktionskurvor . . . . . . . . . .
2.5.2 Partiell integration . . . . . . . . . . . . . . .
2.6 Rationell integrand . . . . . . . . . . . . . . . . . . .
2.6.1 Integration av rationell funktion . . . . . . . .
2.7 Generaliserad integral . . . . . . . . . . . . . . . . . .
2.7.1 Definition och inledande exempel . . . . . . .
2.7.2 Generaliserad integral forts . . . . . . . . . . .
2.8 Volymberäkning . . . . . . . . . . . . . . . . . . . . .
2.8.1 Volym av vissa kroppar . . . . . . . . . . . . .
2.9 Volym och Areaberäkning . . . . . . . . . . . . . . .
2.9.1 Volym, skalmetoden . . . . . . . . . . . . . .
2.9.2 Area av yta . . . . . . . . . . . . . . . . . . .
2.9.3 Kurva och kurvlängd . . . . . . . . . . . . . .
2.10 Tyngdpunkt, tröghetsmoment m.m. . . . . . . . . . . .
2.10.1 Tyngdpunkt . . . . . . . . . . . . . . . . . . .
2.11 Lite om kurvor i planet . . . . . . . . . . . . . . . . .
2.11.1 Parametrisering av en kurva . . . . . . . . . .
5
5
5
6
.
.
.
.
.
.
.
.
.
.
.
.
.
.
.
.
.
.
.
.
.
.
.
.
.
.
.
.
.
.
.
.
.
.
.
.
.
.
.
.
.
.
.
.
.
.
.
.
.
.
.
.
.
.
.
.
.
.
.
.
.
.
.
.
.
.
.
.
.
.
.
.
.
.
.
.
.
.
.
.
.
.
.
.
.
.
.
.
.
.
.
.
.
.
.
.
.
.
.
.
.
.
.
.
.
.
.
.
.
.
.
.
.
.
.
.
.
.
.
.
.
.
.
.
.
.
.
.
.
.
.
.
.
.
.
.
.
.
.
.
.
.
.
.
.
.
.
.
.
.
.
.
.
.
.
.
.
.
.
.
11
11
11
13
14
19
19
19
20
20
21
21
23
23
26
27
27
28
30
30
33
33
39
41
41
42
42
42
42
44
44
48
48
3 Differentialekvation
3.1 Differentialekvation (DE) . . . . . . . . . . . . . . . . . .
3.1.1 Linjär DE med konstanta koefficienter av ordning 1
3.1.2 Olika typer av DE . . . . . . . . . . . . . . . . .
3.2 De av ordning 2 . . . . . . . . . . . . . . . . . . . . . . .
.
.
.
.
.
.
.
.
.
.
.
.
.
.
.
.
51
51
51
55
56
3
.
.
.
.
.
.
.
.
.
.
.
.
.
.
.
.
.
.
.
.
.
.
.
.
.
.
.
.
.
.
.
.
INNEHÅLL
4
3.2.1
3.2.2
Linjär DE med konstanta koefficienter av ordning 2 . . . .
Inhomogen Linjär DE av andra ordningen . . . . . . . .
56
58
4 Taylorutvecklingar
4.1 Taylorpolynom m.m. . . . . . . . . . . . . . . . . . . . .
4.1.1 Fakultet . . . . . . . . . . . . . . . . . . . . . . .
4.1.2 Några funktioners kurvor och dito Taylorpolynom
4.1.3 Hur man bestämmer ett sådant polynom . . . . . .
4.2 Fler Maclaurinserier . . . . . . . . . . . . . . . . . . . . .
4.2.1 Lite knep för att bestämma Taylorutveckling m.m.
4.3 Fyra satser . . . . . . . . . . . . . . . . . . . . . . . . . .
4.4 Omskrivning av resttermen . . . . . . . . . . . . . . . . .
4.4.1 Lagranges form . . . . . . . . . . . . . . . . . . .
4.4.2 Ordoform . . . . . . . . . . . . . . . . . . . . . .
4.5 Maclaurinutveckling av sinus och cosinus . . . . . . . . .
4.5.1 Sinus . . . . . . . . . . . . . . . . . . . . . . . .
4.5.2 Cosinus . . . . . . . . . . . . . . . . . . . . . . .
.
.
.
.
.
.
.
.
.
.
.
.
.
.
.
.
.
.
.
.
.
.
.
.
.
.
.
.
.
.
.
.
.
.
.
.
.
.
.
.
.
.
.
.
.
.
.
.
.
.
.
.
65
65
65
65
66
68
70
70
71
71
72
73
73
73
5 Talföljd och serie
5.1 Talföljd och serie, inledande exempel
5.1.1 Kvotkriteriet . . . . . . . . .
5.1.2 Rotkriteriet . . . . . . . . . .
5.2 Potensserier . . . . . . . . . . . . . .
.
.
.
.
.
.
.
.
.
.
.
.
.
.
.
.
75
75
78
80
81
.
.
.
.
.
.
.
85
85
85
85
89
89
91
92
.
.
.
.
.
.
.
.
.
.
.
.
6 Numerisk beräkning av bestämd integral
6.1 Bakgrund . . . . . . . . . . . . . . . . . .
6.2 Integral som area . . . . . . . . . . . . . .
6.2.1 Grundidön för berökning av integral
6.3 Numerisk integration . . . . . . . . . . . .
6.3.1 Rektangelmetoden . . . . . . . . .
6.3.2 Sekantmetoden . . . . . . . . . . .
6.3.3 Simpsons formel . . . . . . . . . .
.
.
.
.
.
.
.
.
.
.
.
.
.
.
.
.
.
.
.
.
.
.
.
.
.
.
.
.
.
.
.
.
.
.
.
.
.
.
.
.
.
.
.
.
.
.
.
.
.
.
.
.
.
.
.
.
.
.
.
.
.
.
.
.
.
.
.
.
.
.
.
.
.
.
.
.
.
.
.
.
.
.
.
.
.
.
.
.
.
.
.
.
.
.
.
.
.
.
.
.
.
.
.
.
.
.
.
.
.
Kapitel 1
Summa
1.1 Summasymbolen med exempel
Syftet med nedanstående, är att visa (1.9) på sidan 8.
Ex. vis är
3
12 + 22 + 32 .
∑ k2 = {betyder} = k=1
k=2 k=3
k=1
∑ kallas summasymbol, k kallas löpande, 1 undre och 3 övre index. k löper alltså
genom alla heltal mellan undre index 1 och övre index 3.
1.1.1
Summasymbolen
Definition 1.1
För m ≤ n definieras
n
∑ ak = am + am+1 + am+2 + ... + an−2 + an−1 + an
(1.1)
k=m
menas summan av alla termer ak , där k går genom alla heltal mellan
k = m och k = n.
m är undre index, n övre index och k löpande index.
Om m > n definieras vänsterledet i (1.1) som 0.
Exempel 1.1
Ex.vis är
√
√
√
√
√
k = 1+ 2+ 3+ 4
4
S :=
∑
k=1
och
3
T :=
∑ 2k = 20 + 21 + 22 + 23 .
k=0
Antal termer i dessa summor är 4 − 1 + 1 = 4 respektive 3 − 0 + 1 = 4. Eftersom det är lika många termer i summorna, kan vi skriva termerna under samma
summasymbol. Först förskjuter det löpande indexet i den första summan:
4
∑
√
k=
3
∑
k=1
och sedan skriver vi
k=0
3
S+T =
√
k+1
√
k + 1 + 2k ) .
∑(
k=0
5
KAPITEL 1. SUMMA
6
Exempel 1.2
4
3· ∑
√
k=
k=1
4
∑3
√
k.
k=1
Att kunna multiplicera in ett tal c (här c = 3) i summan eller alternativt bryta ut den
är en av två linearitetsegenskaperna för summor. Den andra är den som illustreras
i föregående exempel.
Teorem 1. 1 Följande Linearitetsegenkaper gäller
n
∑ (ak + bk )
=
k=1
n
n
k=1
k=1
∑ ak + ∑ bk ,
n = 1, 2, ...
n
∑ c ak
k=1
(1.2)
n
= c ∑ ak .
k=1
Kommentarer
• En term i summan ak kan ses som en funktion av det löpande indexet eller
variabeln k. Man kan skriva a(k) "a av k" istället för ak .
• Med funktionsbegreppet och med funktionne x2 =: f (x) kan vi få följden
f (i) = i2 , i = 1, 2, .... En följd skrivs som
∞
( f (i))∞
i=1 eller (ak )k=1
(1.3)
mer om följd senare i samband med serie.
• En summa kan då skrivas
n
∑ f ( j) .
j=1
1.1.2 Tre summor
n
S(n) :=
∑ (2k − 1) = n2 .
k=1
Detta kan man bevisa och/eller motivera. Vi börjar med att motivera (1.4) och
därefter visar vi (1.5).
Exempel 1.3
1.1. SUMMASYMBOLEN MED EXEMPEL
7
Vi ser i figuren, att 1 + 3 + 5 = 32 , summan av de 3 första udda positiva heltalen är
lika med 32 . P.s.s. förstår man att summan av de n första udda positiva heltalen är
lika med n2 .
Alla udda heltal kan skrivas 2k − 1, där k är ett heltal. Vidare är 1 = 2 · 1 − 1,
3 = 2 · 2 − 1 och 5 = 2 · 3 − 1. Ex.vis är 11 = 2 · 6 − 1 och 11 är det 6:e udda
positiva heltalet. Summan
50
1 + 3 + +5 + ... + 97 + 99 =
∑ (2k − 1) = 502 = 2500 ,
k=1
eftersom 99 = 2 · 50 − 1 är det 50 :e udda positiva heltalet. Vi får ett mer allmänt
resultat, nämligen
1 + 3 + 5 + ... + (2n − 1) =
n
∑ (2k − 1) = n2 ,
n = 1, 2, 3, ...
(1.4)
k=1
Exempel 1.4
För att beräkna
n
S1 (n) :=
∑ k = 1 + 2 + 3 + 4 + ... + (n − 1) + n ,
k=1
utnyttjar vi (1.4).
n2 = 1 + 3 + 5 + ... + (2n − 1) = (2 · 1 − 1) + (2 · 2 − 1) + (2 · 3 − 1) + ...
+(2(n − 1) − 1) + (2n − 1) = 2 · (1 + 2 + 3 + ... + (n − 1) + n) −1 − 1 − 1, , , , −1 − 1 =
{z
}
|
n termer
2 S1 (n) − n ⇐⇒ n2 + n = 2S1 (n) ⇐⇒
S1 (n) =
Exempel 1.5
n
S1 (n)
n2 + n n(n + 1)
=
.
2
2
(1.5)
Vi beräknar S1 (n) för några olika n m.h.a. identiteten i (1.5).
1
1 · (1 + 1)
1=
2
2
2 · (2 + 1)
1+2 =
2
3
3 · (3 + 1)
1+2+3 =
2
4
1+2+3+4 =
4 · (4 + 1)
2
Vi skall härleda formler för
1 + 2 + 3 + ... + (n − 2) + (n − 1) + n =
n
∑ k =: S1 (n) och
(1.6)
k=1
12 + 22 + 32 + ... + (n − 2)2 + (n − 1)2 + n2 =
n
∑ k2 =: S2 (n) .
k=1
(1.7)
KAPITEL 1. SUMMA
8
där vi redan taget fram formel för (1.6). Vi tar fram en formel för (1.6) genom att
summera två sådana summor så här.
2 S1 (n) = 1 + 2 + 3 + ... + (n − 2) + (n − 1) + n+
+ n + (n − 1) + (n − 2) + ... + 3 + 2 + 1 =
= (n + 1)n ⇐⇒
(n + 1)n 1 2 1
= n + n.
2
2
2
Vi visar nu (1.8) men på ytterligare ett sätt.
S1 (n) =
S2 (n) − S2 (n − 1) = n2 =
( 2
)
k − (k − 1)2 =
n
∑
k=1
2 S1 (n) − n som ger S1 (n) =
(1.8)
n
∑ (2k − 1) =
k=1
1 2 1
n + n.
2
2
Vi ser att vi också har bevisat (1.4). För att få en formel för (1.7), börjar vi med att
n
sätta S3 (n) =
∑ k3 och använder den senare tekniken ovan. Vi får att
k=1
S3 (n) − S3 (n − 1) =
n
∑
( 3
)
k − (k − 1)3 = n3 =
k=1
n
∑ (3k2 − 3k + 1) = 3 S2 (n) − 3 S1 (n) + n .
k=1
Av detta följer att
S2 (n) =
) 1
1( 3
1
1
1
n + 3S1 (n) − n = n3 + n2 + n − n
3
3
2
2
3
alltså att
S2 (n) =
1 3 1 2 1
n + n + n.
3
2
6
(1.9)
Kommentarer
• Både (1.8) och (1.9) kan faktorupppdelas.
1
1
n(n + 1)
1
1
1
n(n + 1)(2n + 1)
S1 (n) = n2 + n =
och S2 (n) = n3 + n2 + n =
.
2
2
2
3
2
6
6
• Med
n
Sα (n) =
∑ kα
α = 0, 1, 2, 3, ...
k=1
kan man visa att
n
Sα (n) =
∑ kα = cα,α+1 nα+1 + cα,α nα + cα,α−1 nα−1 + ... + cα,1 n
(1.10)
k=1
för konstanter cα, j , , j = 1, 2, ..., α − 1, α med cα,α+1 =
För α = 3 är
1
1
och cα,α = .
α+1
2
1
n4 n3 n2
Sα (n) = S3 (n) = n2 (n + 1)2 ≡ + + ,
4
4
2
4
där vi ser att c3,4 =
1
1
och c3,3 = .
4
2
1.1. SUMMASYMBOLEN MED EXEMPEL
• Speciellt ser vi att
9
S3 (n) = [S1 (n)]2 .
• Observera att cα+1,α och cα,α stämmer för α = 1 och α = 2.
Övningar
n
1.1
Ta fram en formel för
∑ k3 = S3 (n).
k=1
1.2
Summan
n−1
n−1
k=0
k=0
∑ xk =: G(x, n) är en geometrisk summa. Man kan visa att
1 − xn
, om x ̸= 1.
1−x
∑ xk =
(a) Hur många termer innehåller summan?
(b) Beräkna G(2, n).
(c) Beräkna G(1/2, n). Vad blir gränsvärdet av denna summa, då n → ∞?
(d) G(x, n) är inte definierad för x = 1. Beräkna gränsvärdet och tolka resultatet
av gränsvärdet
lim G(x, n) .
x→1
1.3
En fotbollsturnering har n + 1 deltagande lag. Lagen möts två och två en gång
(enkelturnering). Hur många matcher spelas totalt i turneringen?
1
i (1.10).
α+1
1.4
*) Bevisa att cα,α+1 =
1.5
Bevisa formlerna (1.8) och (1.9) för S1 (n) respektive S2 (n) med induktion.
10
KAPITEL 1. SUMMA
Kapitel 2
Integral
2.1 Integralens definition och enkla exempel
2.1.1
Integral m.h.a. summor
1
y= f HxL=x2
1
∫ 1
x2 dx
0
Vi beskriver idén med funktionen f (x) = x2 , som integreras över intervallet {x :
0 ≤ x ≤ 1} = [0, 1]. Summan
n
1
1
1
∑ k2 = 3 n3 + 2 n2 + 6 n
k=1
härleds på sidan 8.
Vi får alltså en undersumma till integralen som är
Un =
n−1
n−1
k=0
k=0
1
n−1
1
∑ f (k/n)∆x = ∑ f (k/n) n = ∑ (k/n)2 n =
k=0
(
)
1 n−1 2
1
(n − 1)3 (n − 1)2 n − 1
= 3 ∑k = 3·
+
+
=
n k=0
n
3
2
6
(1 − 1/n)3 (1 − 1/n)2 1 − 1/n
1
+
+
→
3
2n
6n2
3
11
KAPITEL 2. INTEGRAL
12
1
y= f HxL=x2
1
Undersumma
Vi får p.s.s. en översumma till integralen som är
n
Ön =
∑
n
1
1
=
(k/n)2 =
f
(k/n)
∑
∑
n k=1
n
k=1
n
f (k/n)∆x =
k=1
)
( 3
1 n 2
n2 n
1
1
1
1
n
1
= 3 ∑k = 3·
+ +
= + + 2→
n k=1
n
3
2 6
3 2n 6n
3
1
y= f HxL=x2
1
En översumma
2.1. INTEGRALENS DEFINITION OCH ENKLA EXEMPEL
13
Givet en begränsad funktion f (x) på ett kompakt intervall [a, b] och en indelning av [a, b] i delintervall [xk−1 , xk ], k =
1, 2, ..., n. Låt vidare uk ≤ f (x) och ök ≥ f (x) för xk−1 ≤ x ≤ xk för
k = 1, 2, 3, ..., n. Då är
Definition 2.1
Un :=
n
n
k=1
k=1
∑ (xk − xk−1 ) · uk och Ön = ∑ (xk − xk−1 ) · ök
(2.1)
en undersumma respektive översumma till f (x) på intervalllet [a, b].
Kommentarer
• Observera att Un ≤ Ön för alla under- och översummor. Det gäller t.o.m. att
Um ≤ On , m, n = 1, 2, 3, ....
• Att precis ett tal, i detta fall talet 1/3 ligger mellan alla dessa under- och
∫ 1
översummor, säger att integralen
x2 dx existerar och antar värdet 1/3.
0
• Även om man inte har lika långa delintervall ∆x = 1/n, så kan man visa att
det bara finns ett tal mellan alla under- och översummor, nämligen talet 1/3.
• Funktionen f (x) är integrerbar över intervallet [0, 1], i Riemanns mening.
∫
∫
• I ab f (x)dx, så kallas för integraltecken, a undre gräns, b övre gräns, f (x)
integrand, x är integrationsvariabel och dx differential.
Givet en begränsad funktion f (x) på de kompakta intervalllet [a, b]. Om det finns precis ett tal I mellan alla under- och översummor, d.v.s.
U ≤I≤Ö
Definition 2.2
för alla undersummor U och översumor Ö, så säger man att f (x) är
integrerbar (i Riemanns mening) på intervallet [a, b]. Man betecknar
integralen
∫ b
I=
f (x) dx
(2.2)
a
Teorem 2. 1 Om f (x) är kontinuerlig på [a, b], så är f (x) integrerbar
där.
2.1.2
Integralkalkylens medelvärdessats I
Teorem 2. 2 Antag att f (x) är kontinuerlig på intervallet [a, b] (a < b).
Då finns ett ξ : a ≤ ξ ≤ b, sådant att
∫ b
a
Bevis:
f (x) dx = (b − a) f (ξ).
(2.3)
KAPITEL 2. INTEGRAL
14
f (x) antar ett minsta värde och ett största värde fmin respektive fmax , enligt Satsen
om minsta och största värde,d.v.s.
fmin ≤ f (x) ≤ fmax .
Då är
(b − a) fmin ≤
∫ b
a
f (x)dx ≤ (b − a) fmax
eftersom VL och HL är under- respektive översumma. Genom att dividera med
b − a i samtliga led erhålls
fmin ≤
1
b−a
∫ b
a
f (x)dx =: y ≤ fmax .
Enligt Satsen om mellanliggande värde, finns ξ ∈ [a, b] sådant att f (ξ) = y, d.v.s.
f (ξ) =
1
b−a
∫ b
f (x) dx .
a
Genom att multiplicera med b − a i båda led följer likheten.
2.1.3
Integralkalkylens huvudsats (5.5)
För en kontinuerlig funktion f (x) på intervallet [a, b] gäller
∫ b
∫ c
f (x)dx =
a
∫ b
f (x)dx +
a
f (x)dx
(2.4)
c
för a ≤ c ≤ b.
Teorem 2. 3 Vi definierar
∫ x
F0 (x) =
f (t)dt
(2.5)
a
för en kontinuerlig funktion f . Då är F0′ (x) = f (x).
Bevis:
y
y= f HxL
f HxL
a
x, x+D x
b
x
2.1. INTEGRALENS DEFINITION OCH ENKLA EXEMPEL
15
∫ x
Ur figuren definierar vi "areafunktionen"
a
f (t)dt =: F0 (x), som arean mellan
y = 0, y = f (x), x = a och x = x. Vi ser att
F0 (x + ∆x) − F0 (x) =
∫ x+∆x
f (t)dt = f (ξ)∆x
x
enligt Medelvärdessatsen, så att
F0 (x + ∆x) − F0 (x)
= f (ξ) .
∆x
Om nu ∆x → 0, så kommer f (ξ) → f (x), p.g.a. kontinuitet. Alltså har även HL
ett gränsvärde och det måste vara F0′ (x) eftersom mittenledet är differenskvoten för
f (x), d.v.s.
F0 (x + ∆x) − F0 (x)
f (x) = lim
= F0′ (x)
(2.6)
∆x→0
∆x
En funktion F(x), sådan att F ′ (x) = f (x) kallas primitiv
funktion till f (x).
Definition 2.3
Vi behöver en hjälpsats innan vi bevisar nästa sats.
Teorem 2. 4 Om F1 (x) och F2 (x) är primitiva funktioner till f (x) på
intervallet [a, b], så är F1 (x) = F2 (x) +C för någon konstant C.
Bevis:
Bilda funktionen G(x) := F1 (x) − F2 (x). Vi visar att G(x) är konstant. Vi har att
G′ (x) = f (x) − f (x) ≡ 0 .
Tag två olika x−värden, x1 och x2 i I. Vi visar att G(x1 ) ̸= G(x2 ) är omöjligt. Då
finns, enligt medelvärdessatsen, ξ mellan x1 och x2 , sådant att
G(x1 ) − G(x2 ) = (x1 − x2 ) g(ξ)
men g(ξ) = 0 och x1 − x2 ̸= 0, så att G(x1 ) = G(x2 ).
Kommentarer
• I själva verket har vi ekvivalens:
F1′ (x) = F2′ (x) ⇐⇒ F1 (x) +C = F2 (x)
för någon additiv konstant C.
Låt nu F(x) vara en primitiv
∫ funktion till f (x) i (2.5). Med konstruktionen av areax
funktionen som F0 (x) =
a
f (t)dt, ser vi att F0 (x) också är en primitiv funktion
till f (x). Då är alltså F(x) +C = F0 (x) för någon konstant C. Vi sätter x = a. Då
är F0 (a) = 0 = F(a) +C, så att C = −F(a). Därmed är F0 (x) = F(x) − F(a). Sätt
nu x = b. Då får vi F0 (b) = F(b) − F(a), d.v.s.
KAPITEL 2. INTEGRAL
16
Teorem 2. 5 om f (x) är en kontinuerlig funktion i intervallet [a, b] och
F(x) en primitiv funktion till f (x), så är
∫ b
a
f (x)dx = [F(x)]ba = F(b) − F(a) .
(2.7)
Kommentarer
1. Begreppen under- och översumma fungerar även om f (x) < 0, helt eller
delvis i integrationsintervallet.
2. Om integranden f (x) < 0, bidrar detta med en "area" < 0. Integral är "area
med tecken".
3. Ett intervall, såsom [a, b] är slutet och begränsat och kallas kompakt. Man
kan bevisa att en kontinuerlig funktion f (x) är integrerbar. För detta krävs
ett axiom.
4. En funktion F(x), sådan att F ′ (x) = f (x) kallas alltså primitiv funktion till
x3
en primitiv funktion till f (x) = x2 . Även F1 (x) =
f (x). Ex.vis är F(x) =
3
x3
− 2 är det. I själva verket ges alla primitiva funktioner till f (x) = x2 av
3
x3
+C, där C är en konstant.
3
∫ b
5.
f (x) dx kallas bestämd integral.
(a)
∫a
f )x) dx kallas obestämd integral och betyder alla primitiva funk-
(b)
tioner till f (x).
∫
6. Alltså är
f (x) dx = F(x) +C, där F(x) är en primitiv funktion till f (x) och
C en godtycklig konstant.
n−1
Exempel 2.1
Givet summan S(n) :=
1
∑ n + j2 /n .
Är det en under- eller över-
j=0
summa till någon integrand? Vilken är i så fall integranden och x−gränserna?
Beräkna också (i så fall) lim S(n).
n→
Lösning: Summanden (d.v.s. en term) är
1
1
1
= ·
2
n + j /n n 1 + ( j/n)2
1
. Vi observerar att j = 0, 1, 2, ..., n−
1 + x2
1 och med delintervall med gränser x j = ( j)/n och x j+1 = ( j +1)/n, j = 0, 1, 2, ..., n−
1. I intervallet med gränser x j och x j+1 är summanden f ( j/n) ≥ f (x). Alltså är
1
summan en översumma till integralen Int01
dx. En primitiv funktion till f (x)
1 + x2
är F(x) = arctan x. Alltså är
där vi har brutit ut 1/n = ∆x och får f (x) =
n−1
1
1
1
lim ∑
= lim ·
=
2
n→∞
n→∞
n 1 + ( j/n)2
j=0 n + j /n
∫ 1
0
1
π
dx = [arctan x]10 =
2
1+x
4
2.1. INTEGRALENS DEFINITION OCH ENKLA EXEMPEL
∫ 1
Exempel 2.2
Beräkna integralen
−1
17
x2 dx.
Lösning: Vi utnyttjar att integranden är
en jämn funktion. Med f (x) = x2 kan detta
uttryckas som att f (−x) = (−x)2 = x2 =
f (x). med ex.vis med dito primitiv funkx3
tion F(x) =
blir integralen
3
[ 3 ]1
∫ 1
∫ 1
2
x
2
2
= ... = .
x dx = 2
x dx = 2
3 0
3
−1
0
y
y=x2
1
-
1
I föregående exempel är integranden jämn och integrationsintervallet symmetriskt
(kring origo).
∫ π/4
Exempel 2.3
Beräkna integralen
−π/4
sin5 x dx.
Lösning: Även om vi kanske inte kan ta fram en primitiv funktion till integranden sin5 x, kan vi ändå beräkna integralen. Integranden är udda och integrationsintervallet symmetriskt. Att den är udda innebär att
g(−x) := sin5 (−x) = (− sin x)5 = − sin5 x .
Då är integralen = 0.
Exempel 2.4
Beräkna integralen...
∫ 3
dx
1
Exempel 2.5
x2
∫ 3
=
1
[ ]3
1
1 1 2
x dx = −
= − = .
x 1 1 3 3
−2
Givet funktionen e−x/2 =: f (x).
∫ 2
f (x)dx.
(a) Beräkna
0
(b) Bestäm alla primitiva funktioner till f (x).
(c) Bestäm en primitiv funktion till f (x).
x
KAPITEL 2. INTEGRAL
18
Lösning:
(a) En primitiv funktion F(x) till f (x) är F(x) = −2 · e−x/2 . Detta ser man
genom att derivera funktionen F(x) = −2 · e−x/2 :
F ′ (x) = −2 ·
∫ 2
0
e|−x/2
· (−1/2) = {och alltså} = f (x) .
{z }
| {z }
yttre derivata inre derivata
(
)
[
]2 [
]2
1
−x/2
−x/2
−2/2
−0/2
f (x)dx = −2 · e
= −2e
= −2(e
−e
)=2 1−
.
0
0
e
(b) Alla primitiva funktioner till f (x) är
∫
e−x/2 dx = −2e−x/2 +C där C är en godtycklig konstant.
(c) En primitiv funktion till f (x) är ex.vis F(x) := 2(1 − e−x/2 ).
Kommentarer
• Det gäller att kunna skilja på de tre olika frågorna i exemplet ovan.
• Vi ser att det är bra att (I) skriva om integralen och/eller integranden innan
integrationen
• och därefter (II) ta fram en primitiv funktion och till sist
• (III) förenkla/skriva om svaret.
• Vi kan lösa (a) genom att byta plats på undre och övre gräns, och byta tecken
på den primitiva funktionen.
(
)
∫ 2
[
]0
[
]2
1
−0/2
−2/2
−x/2
−x/2
= 2(e
−e
) = 2 1−
=2 e
,
f (x)dx = −2 · e
2
0
e
0
• Alternativt kan man byta tecken redan på integranden och därmed plats på
gränserna (Jämför med förra punkten).
∫
Exempel 2.6
Beräkna
dx
.
x
Lösning: Integranden är x−1 , en potensfunktion. En primitiv funktion borde vara
x−1+1
men detta fungerar bara om exponenten ̸= −1. I just detta fall är exponen−1 + 1
ten −1.
En primitiv funktion är då ln |x|.
∫
dx
Svar:
= ln |x| +C, där C är en godtycklig konstant.
x
Kommentarer
• Man kan göra följande omskrivning.
∫
dx
= ln |x| +C = ln |x| + ln(eC ) = ln |k x|
x
där k = eC .
• Man kan inte integrera denna funktion över en mängd, såsom [−1, 1], efter1
som f (x) = inte är definierad för x = 0 (en "singularitet").
x
2.2. PRIMITIV FUNKTION TILL OLIKA FUNKTIONSKLASSER
2.1.4
19
Integral tolkad som area
Om det är möjligt att tolka området mellan funktionskurva och x−axel, med a ≤
x ≤ b, som en känd yta, såsom en del av en cirkelskiva, kan integralen beräknas
rent geometriskt.
Exempel 2.7
Beräkna
∫ √2 √
0
2 − x2 dx.
Lösning: Vi har kurvan
y
1.4
1.2
1.0
0.8
0.6
0.4
0.2
0.2
0.4
0.6
0.8
Ytan är en kvarts cirkelskiva med radie r =
A=
1.0
1.2
1.4
x
√
2. Arean är alltså
1
π
· π r2 = a.e.
4
2
2.2 Primitiv funktion till olika funktionsklasser
2.2.1
Primitiv funktion till/integral av potensfunktion
Mer allmänt är


xn+1


∫
 n + 1 +C, n ̸= −1
n
x dx =



ln |x| +C, n = −1
Exempel 2.8
∫
x + 5x2
√ dx =
x
∫ (
√
∫
∫
)
x + 5x3/2 dx = x1/2 dx + 5 x3/2 dx =
=
x5/2
2x3/2
2x3/2
+5·2·
+C =
+ 2 · x5/2 +C =
3
5
3
=
2 3/2
x (3x + 1) +C
3
där C är en godtycklig konstant.
(2.8)
KAPITEL 2. INTEGRAL
20
2.2.2
Integral av exponentialfunktion
Vi har redan integrerat en exponentialfunktion i exempel 2.5.
Exempel 2.9
Bestäm en primitiv funktion till (a) f (x) := ek x ,
(b) g(x) := 3 · 2x .
Lösning: (a) Vi antar först att k ̸= 0.
∫
1
1
f (x)dx = ekx +C så att en primtiv funktion är F(x) = ekx .
k
k
För k = 0 är f (x) = 1, så att en primitiv funktion är F(x) = x.
(b) Vi skriver om funktionen så att basen blir e.
3 · 2x = 3 · ex ln 2
där ln 2 = k som i (a), så att en primitiv funktion är
2.2.3
3
· 2x .
ln 2
Integral av trigonometriska funktioner
Exempel 2.10
Funktion
cos x
tan x
En primitiv funktion
sin x
1
− cos 2x
2
− ln | cos x|
1
1 + tan2 x ≡
cos2 x
1 + cos 2x
cos2 x ≡
2
tan x
sin 2x x
+
4
2
sin 2x
Exempel 2.11
Bestäm en primitiv funktion till cos3 x =: h(x).
Lösning:
cos3 x = cos2 x · cos x = (1 − sin2 x) cos x .
Vi ser att cos x är derivatan till sin x . En primitiv funktion ges av
H(x) = sin x −
sin3 x
,
3
ty när vi deriverar H(x) får vi
sin2 x
· cos x = (1 − sin2 x) cos x .
3
Att det ta fungerar beror på att den inre derivatan är cos x och cos x finns som faktor
till integranden.
H ′ (x) = cos x − 3 ·
I princip klarar man att integrera/bestämma primitiv funktion till alla udda potenser
av sin x och cos x på liknande sätt. Om en jämn potens av sin x eller cos x är integrand, klarar man det med trigonometriska omskrivningar, ex.vis
1 + cos 2x
cos2 x =
.
2
2.3. DERIVATA OCH INTEGRAL
2.2.4
21
Linearitetsegenskaper hos integralen
Teorem 2. 6 Dessa egenskaper är
∫
∫
( f (x)+g(x))dx =
∫
∫
f (x)xdx+
g(x)dx och
∫
k f (x)dx = k
f (x)dx
(2.9)
om f (x) och g(x) är kontinuerliga. och gäller även för bestämd integral.
Kommentarer
• Likheterna för obestämd integral tolkas som "likhet sånär som på en additiv
konstant.".
• För att bevisa den första identiteten (likheten), låt F(x) och G(x) vara primitiva funktioner till f (x) respektive g(x). VL är då alla primitiva funktioner
till f (x) + g(x) och kan skrivas F(x) + G(x) +C.
HL är F(x) +C1 + G(x) +C2 .
Det är klart att VL och HL, d.v.s.
F(x) + G(x) +C och F(x) +C1 + G(x) +C2
är lika sånär som på en konstant.
2.3 Derivata och integral
x3
en primitiv funktion. OM vi deriverar
3
F(x) får vi tillbaka f (x). Om vi integrerar
Exempel 2.12
Med f (x) = x2 är F(x) =
∫ x
F(x) :=
a
[ 3 ]x
t
x3 − a3
t dt =
=
3 a
3
2
och därefter deriverar får vi
d
d
F(x) =
dx
dx
Exemplet ovan visar att
d
dx
Exempel 2.13
d
dx
∫ a
x2
∫ x
t 2 dt =
a
d x 3 − a3
= x2 .
dx 3
∫ x
f (t)dt = f (x) .
a
f (t)dt = −
d
dx
∫ x2
a
f (t)dt = − f (x2 ) · 2x .
(2.10)
KAPITEL 2. INTEGRAL
22
Kommentarer
• vi har alltså att
• Om deriverar först och sedan integrerar:
∫
d
f (x) dx =
dx
∫
f ′ (x)dx = f (x) +C .
(2.11)
2.4. INTEGRALER FORTS
23
2.4 Integraler forts
2.4.1
Variabelsubstitution
Exempel 2.14
Integralen
∫
1
sin 3x dx = − cos 3x +C .
3
Denna kompensation för den inre funktionens derivata (Här = 3) har vi gjort tidigare.
Man kan också "substituera", som innebär en "omskalning" med ett byte av vari1
abel. Vi byter: 3x = t. Då är x = · t = x(t), d.v.s. vi kan se x som en funkion av
3
variabeln t. Genom att derivera får vi
x′ (t) ≡
dx 1
dt
= ⇔ dx = .
dt
3
3
Vi byter konsekvent x mot t i integralen:
∫
∫
sin 3x dx =
sint
dt
1
=
3
3
∫
1
1
sint dt = − cost +C = − cos 3x +C ,
3
3
där vi i sista ledet har bytt tillbaka till x.
För att lösa en bestämd integral, med samma integrand ex.vis
∫ π/6
sin 3x dx
0
kan man bara sätta in dessa "x−gränser" i den primitiva funktionen, alltså
[
]π/6
[
]0
∫ π/6
1
1
1
1
sin 3x dx = − cos 3x
= {knep}
cos 3x
= (cos 0−cos(3·π/6))) = .
3
3
3
3
0
0
π/6
Man kan också byta till t−gränser. Variabelsubstitution förutsätter att x är en funktion av t. I stället för att derivera, så differentierar man. I exemplet ovan innebär
det att
t
dt
x = =⇒ dx = .
3
3


3x = t
⇒ dx = dt/3




∫ π/6


=
sin 3x dx =
x − gränser 0 π/6


0




t − gränser 0 3 · π/6 = π/2
=
1
3
∫ π/2
sint dt =
0
1
1
1
π/2
[− cost]0 = (cos 0 − cos(π/2)) = .
3
3
3
Exempel 2.15
∫
∫
3
sin xdx =
∫
=
(1 − cos2 x) sin x dx = {cos x = t → dt = − sin xdx} =
(1 − t 2 )(−dt) =
∫
(t 2 − 1)dt =
t3
cos3 x
− t +C =
− cos x +C .
3
3
KAPITEL 2. INTEGRAL
24
Vi visar i följande exempel, att vid ett variabelbyte (subst.) räcker det med att den
"gamla" variabeln (här x) är en funktion av den nya variabeln.
∫ 5
Exempel 2.16
Beräkna integralen
1
xdx med substitutionen x = x(t) = t 2 −4t +5.
Lösning: Vi får att dx = (2t −4)dt. Gränsbyten: Med x = 1 får vi x(t) = t 2 −4t +
5 = 1 ⇔ t = 2. Med x = 5 får vi x(t) = t 2 − 4t + 5 = 5 ⇔ t = 0 eller t = 4. Vilken
av de två övre x−gränserna skall man ta? Duger båda? Vi beräknar motsvarande
t−integral med de båda övre t−gränserna.
∫ 5
∫ 0
xdx =
1
2
(t 2 − 4t + 5)(2t − 4)dt = ... = 12 ,
men även med den övre t−gränsen t = 4 ger
∫ 5
∫ 4
xdx =
1
Exempel 2.17
2
(t 2 − 4t + 5)(2t − 4)dt = ... = 12 .
x
Bestäm en primitiv funktion till g(x) = √
.
1 − x2
√
1 − x2 = t. Här är t en funktion av x och inte vice versa.
√
1
2t
tdt
dx
= √
·(−2t) = − √
⇔ dx = − √
.
1−x2 = t 2 ⇒ x = 1 − t 2 ⇒
2
2
dt 2 1 − t
2 1−t
1 − t2
Lösning: Sätt
Insatt i x−integralen får vi
∫
∫ √
∫
√
tdt
x
1 − t2
√
·√
dx = −
= − dt = −t +C = C − 1 − x2 .
t
1 − x2
1 − t2
1
(a) Vi deriverar arcsin x och får √
.
1 − x2
1
(b) Bestäm nu en primitiv funktion till √
. Vi ser att uttryckena i (a) och
2x − x2
(b) innanför rottecknena liknar varandra. Vi kvadratkompletterar under rottecknet.
Exempel 2.18
1
1
1
√
=√
=√
2
2
2x − x
1 − (1 − 2x + x )
1 − (x − 1)2
så att
∫
∫
dx
√
=
2x − x2
∫
√
dx
1 − (x − 1)2
= {x − 1 = t ⇔ x = t + 1 ⇒ dx = dt} =
dt
√
= arcsint +C = arcsin(x − 1) +C .
1 − t2
1
Svar: En primitiv funktion till √
är arcsin(x − 1).
2x − x2
2.4. INTEGRALER FORTS
25
√
Vi deriverar nu F(x) = ln |x + x2 + 1|.
Exempel 2.19
Lösning:
√
(
)
1
2x
1
x2 + 1 + x
1
√
√
F (x) =
· 1+ √
= {MGN}
· √
=√
.
x + x2 + 1
2 x2 + 1
x + x2 + 1
x2 + 1
x2 + 1
′
De två sista exemplen ger oss möjlighet att integrera vissa rotfunktioner.
Vi vet att D arctan x =
Exempel 2.20
D arctan(x/a) =
1
x2 + 1
, så att
1
1
a
· = 2
2
1 + (x/a) a a + x2
Integralen
∫ 3
0
1
dx
= {a = 3} =
x2 + 9
3
∫ 3
0
3dx
1
= [arctan(x/3)]30 =
x 2 + 32 3
1
1 π
π
(arctan 1 − arctan 0) = · =
.
3
3 4 12
Denna integral kan man även lösa genom V.S.
∫ b
Definition 2.4
Givet den bestämda integralen
a
f (x) dx med a ≤ b.
En funktion x = x(t) är en variabelsubstitution, om x = x(t) : [α, β] 7→
[a, b], x(α) = a, x(β) = b och x′ (t) är kontinuerlig. Om α > β beaktar
man istället intervallet [β, α].
Teorem 2. 7
∫ b
∫ β
f (x) dx =
a
α
f (x(t))x′ (t)dt
(2.12)
Kommentarer
• Om inte x = x(t) är en funktion av t på intervallet [α, β], så är inte likheten i
(2.12) garanterad.
• Motsvarande likhet finns för obestämd integral och bevisas genom att derivera båda led m.a.p. t:
VL:
och
HL:
d
dt
∫
d
dt
∫
f (x) dx =
dx d
dx
·
=
· f (x)
dt dx
dt
f ((x(t))x′ (t)dt = f ((x(t))x′ (t) = f ((x(t))
dx
.
dt
KAPITEL 2. INTEGRAL
26
2.4.2
V.S. t = tan(x/2)
Denna V.S. fungerar på raionella funktioner i cos x och sin x. Vi får t = tan(x/2) ⇐⇒
2dt
x = 2 arctant +C0 , som ger sambandet dx = 2
. Vi får vidare att
t +1
sin x =
2t
1 − t2
och
cos
x
=
.
1 + t2
1 + t2
(2.13)
Exempel 2.21
(a) Ex.vis blir
∫
dx
=
sin x
∫
1 + t 2 2dt
=
·
2t
1 + t2
∫
dt
= ln |t| +C = ln | tan(x/2)| +C .
t
(b) I integralen nedan kan vi byta gräner
∫ π/2
dx
π/3
sin x
{
=
x − gränser π/3
√ π/2
t − gränser tan(π/6) = 1/ 3 tan(π/4) = 1
∫ 1
=
√
1/ 3
√
dt
1
= [lnt]11/√3 = ln 1 − ln(1/ 3) = ln 3 .
t
2
}
=
2.5. MER OM INTEGRALER
27
2.5 Mer om integraler
2.5.1
Area mellan funktionskurvor
Exempel 2.22
Givet de två funktionerna f (x) = 3 − x och g(x) = 2/x. Bestäm
arean av ytan som begränsas av kurvorna y = f (x) och y = g(x).
Lösning: Vi bestämmer skärningspunkternas x−kooordinater mellan kurvorna.
f (x) = g(x) ⇔ x = 1 eller x = 2
Arean, sånär som på tecken, ges av
∫ 2
1
]2
[
1
3
x2
( f (x)−g(x))dx = 3x − − 2 ln x = 3(2−1)− (22 −12 )−2(ln 2−ln 1) = −2 ln 2 .
2
2
2
1
Detta tal är > 0. Alltså är det arean mellan kurvorna.
y
y=gHxL
y= f HxL
x
x1
x2
Kommentarer
• Om, som i exemplet ovan, gränserna inte är givna, så är det skärningspunkternas x−koordinater som är integrationsgränser. Man behöver inte veta
∫ b
vilken funktion som är störst i intervallet. Att Z :=
a
( f (x) − g(x))dx < 0
betyder att f (x) ≤ g(x) för a ≤ x ≤ b. Man svarar då givetvis med −Z eftersom −Z > 0.
• I vissa uppgifter kan x−gränserna vara givna, och om inte, så räknas de ut
som i exemplet ovan.
• Om två funktionskurvor korsar varandra i det inre av integrationsintervallet,
gäller det att veta vilken funktion, som är störst i respektive delintervall.
• Om en eller båda kurvorna ligger under x−axeln i integrationsintervallet,
ges arean ändå av samma integral, som ovan. Detta inser aman genom att
man "lyfter upp" båda kurvorna lika mycket, så att båda kurvorna hamnar
ovanför x−axeln i integrationsintervallet. Dett gör man genom att addera en
tillräckligt stor konstant C till båda funktionerna. Arean är då
∫ b
A=
∫ b
( f (x)+C)dx−
a
∫ b
(g(x)+C)dx =
a
a
( f (x)+C −(g(x)+C))dx =
∫ b
( f (x)−g(x))dx .
a
KAPITEL 2. INTEGRAL
28
2.5.2
Partiell integration
För att integrera en produkt av funktioner används partiell integration. Man utnyttjar då derivata av produkt.
Betäm en primitiv funktion till (3x + 1)e−x/2 .
Exempel 2.23
Lösning: Detta är en produkt av två funktioner. Vi skall utnyttja det vi vet om
derivata av en produkt, nämlingen
D(u(x)v(x)) = u′ (x)v(x) + u(x)v′ (x) .
Vi integrerar nu båda sidorna och får
∫
u(x)v(x) =
u′ (x)v(x)dx+
∫
u(x)v′ (x)dx ⇔
∫
u′ (x)v(x)dx = u(x)v(x)−
∫
u(x)v′ (x)dx .
Vi får en bekvämare formel/identitet om vi sätter u(x) = F(x) och u′ (x) = F ′ (x) =
f (x), samt v(x) = g(x) . Detta ger
∫
f (x)g(x)dx = F(x)g(x) −
∫
F(x)g′ (x)dx .
(2.14)
Kommentarer
• Termen F(x)g(x) kallas "utintegrerad" term eller del.
• Samma identitet gäller för bestämd integral.
• Några råd: Antag att p(x) är ett polynom.
För
För


cos kx
p(x) · sin kx

 kx
e
, välj p(x) = g(x) .
{
ln(k x)
p(x) ·
arctan(k x)
, välj p(x) = f (x) .
I detta exempel väljer vi alltså g(x) = 3x + 1. Vi får
∫
(3x+1)e−x/2 dx = (3x+1)(−2)e−x/2 −
∫
3(−2)e−x/2 dx = ... = −2e−x/2 (3x+7)+C ,
så att en primitiv funktion är 1 − 2(3x + 7)e−x/2 .
∫ 1
Exempel 2.24
Beräkna integralen
x arctan x dx.
0
2.5. MER OM INTEGRALER
29
Lösning: Vi räknar först utan gränser.
∫
x arctan x dx =
x2
1
arctan x −
2
2
=
x2
1
arctan x −
2
2
∫
∫
x2 + 1
1
dx +
2
x +1
2
x2 ·
∫
1
x2 + 1
1
x2 + 1
dx =
dx =
x 1
x2
arctan x − + arctan x +C .
2
2 2
Motsvarande bestämda integral är alltså
[( 2
)
]1
x
x
1
1
arctan x −
+
= (π − 2) .
2 2
2 0 4
=
Beräkna
Exempel 2.25
Lösning:
∫
1·
x·
2
√
√
∫ √
x2 + 1dx.
x2 + 1 dx
x2 + 1 −
∫
∫ √
= x·
x2 + 1 dx =
x2 + 1 −
(x2 + 1)dx
√
+
x2 + 1
· x2 + 1 dx = x ·
∫ √
√
√
∫
∫
xdx
x· √
=
x2 + 1
dx
√
⇐⇒
x2 + 1
x2 + 1 + ln(x +
√
x2 + 1) +C ⇔
)
√
1( √ 2
x · x + 1 + ln(x + x2 + 1 +C1 .
2
∗ För att integrera eax sin bx, med a ̸= 0 och b ̸= 0, kan man välja
Exempel 2.26
ex.vis sin bx = g(x). Vi får
∫
∫ ax
eax
e
sin bx −
b cos bx dx =
eax sin bx dx =
a
a
( ax
)
∫ ax
eax
e
e
2
=
sin bx −
b cos bx −
(−b sin bx) dx =
a
a2
a2
∫
eax
eax
b2
=
sin bx − 2 b cos bx − 2 eax sin bxdx ⇐⇒
a
a
a
(
)∫
b2
eax
eax
= 1+ 2
eax sin bxdx =
sin bx − 2 b cos bx ⇐⇒
a
a
a
∫
a2
e sin bxdx = 2
a + b2
ax
(
)
eax
eax
sin bx − 2 b cos bx =
a
a
eax
(a sin bx − b cos bx) +C .
a2 + b2
KAPITEL 2. INTEGRAL
30
2.6 Rationell integrand
2.6.1
Integration av rationell funktion
T (x)
, där T (x) och N(x) är polynom, så kallas f (x) en rationell funkN(x)
tion. Ex.vis är
1
f (x) = x2 + 1, g(x) =
2x − 1
Om f (x) =
2
rationella. Polynomet f (x) = x2 + 1 = x 1+1 och alltså en kvot mellan två polynom.
Vi utvecklar en sådan funktion enligt följande schema.
• Om grad T (x) ≥ grad N(x), så polynomdivision (pol.div.)
• Om grad T (x) < grad N(x), så uppdelning i partialbråk (PBU)
För att ex.vis bestämma en primitiv funktion till en rationell funktion måste den
alltså utvecklas.
2x2 + 1
har högre grad på täljaren än på
x−1
nämnaren. Den utvecklas med polynomdivision
Exempel 2.27
En funktion såsom f (x) =
2x2 + 1
:
x−1
2x + 2
x2 + 1 |x − 1
−(2x2 − 2x)
2x + 1
−(2x − 2)
3
2x2 + 1
3
= 2x + 2 +
.
x−1
x−1
Varje term kan nu integreras och därmed hela funktionen. Ex.vis är
)
∫
∫
∫ (
2x2 + 1
3
f (x)dx =
dx =
2x + 2 +
dx = x2 + 2x + 3 ln |x − 1| +C .
x−1
x−1
så att
En primitiv funktion är x2 + 2x + 3 ln |x − 1| .
2x − 3
har lägre grad på täljaren än på nämnaren.
x2 + 3x
Nämnaren kan dessutom faktoruppdelas som x2 + 3x = x(x + 3). Då delar man upp
funktionen g(x) i partialbråk (partialbråksuppdelning, PBU). Man ansätter
Exempel 2.28
Funktionen g(x) =
g(x) =
2x − 3
A
B
A(x + 3) + Bx
= +
= {liknämnigt} =
.
x2 + 3x
x x+3
x(x + 3)
Genom att jämföra och identifiera täljarna i första ledet (VL) och sista ledet (HL)
får vi


VL
HL
A = −1
x:
2 = A+B ⇔


1 : −3 = 3A
B=3
2.6. RATIONELL INTEGRAND
Därmed är g(x) =
∫ 2
1
31
2x − 3
3
1
=
− . Ex.vis är
2
x + 3x x + 3 x
g(x)dx = [3 ln(x + 3) − ln x]21 = (3(ln 5 − ln 4) − (ln 2 − ln 1) = 3 ln 5 − 7 ln 2 .
Detta tal är negativt vilket tyder på att kurvan y = g(x) ligger helt eller delvis under
x−axeln.
2x − 1
i partialbråk, som föregående
x2 + 3x + 2
2
exempel, ser vi att x + 3x + 2 = (x + 1)(x + 2) . Vi ansätter alltså
Exempel 2.29
För att dela upp g(x) :=
2x − 1
x2 + 3x + 2
Detta ger ekvationerna
{
x1 :
x0 :
=
A
B
(A + B)x + 2A + B
+
=
.
x+1 x+2
x2 + 3x + 2
2 = A+B
⇐⇒
−1 = 2A + B
{
A = −3
B=5
5
3
−
. Nu kan man bestämma en primitiv funktion till g(x).
x+2 x+1
)
∫
∫ (
5
3
g(x) dx =
−
dx = 5 ln |x + 2| − 3 ln |x + 1| +C .
x+2 x+1
så att g(x) =
Alltså är en primitv funktion 5 ln |x + 2| − 3 ln |x + 1| .
Exempel 2.30
(a) Bestäm en primitiv funktion till f (x) =
3x + 1
x2 (x + 1)
.
Lösning: Graden av täljaren är 0 < 3, som är graden av nämnaren. Alltså
PBU. Vi ansätter då
Ax + B
C
A B
C
= + 2+
+
2
x
x+1
x x
x+1
eftersom regeln är att täljaren skall vara (minst) en grad lägre än nämnaren.
Ax + B
Nu kan man inte direkt ta fram en primitiv funktion till
men varje
x2
term i HL kan lätt integreras. För att bestämma konstanterna A, B och C gör
vi liknämnigt i HL och jämför täljarna i VL och HL.
f (x) =
=
3x + 1
x2 (x + 1)
=
Ax + B
C
+
=
2
x
x+1
Ax2 + Ax + Bx + B +Cx2
⇔
x2 (x + 1)
x2 :
x1 :
x0 :
VL
HL
0 = A +C
3 = A+B
1 = B
KAPITEL 2. INTEGRAL
32
Vi ser att A = 2, B = 1 och C = −2 . Alltså är
)
(
)
∫
∫ (
2 1
2
x
1
f (x)dx =
+ 2−
dx = 2 ln
− +C.
x x
x+1
x+1
x
(
)
x
1
En primitiv funktion är ex.vis F(x) = 2 ln
− .
x+1
x
(b) Bestäm alla primitiva funktioner till g(x) =
Lösning:
3x + 1
.
x(x + 1)2
Med en liknande ansättning får vi
g(x) =
3x + 1
Ax + B C
=
+
2
x(x + 1)
(x + 1)2 x
Ax + B
i två termer som lätt kan
(x + 1)2
integreras. I stället för vi följande omskrivning.
Men vi kan inte lika lätt dela upp termen
Ax + B
A(x + 1) + B − A
A
B−A
=
=
+
2
2
(x + 1)
(x + 1)
x + 1 (x + 1)2
där varje term lätt kan integreras. Detta visar att ansättningen vid PBU kan
se ut så här (med B − A bytt mot B).
g(x) =
3x + 1
B
A
C
+
=
+ .
2
2
x(x + 1)
x + 1 (x + 1)
x
Liknämnigt och sedan genom att sätta täljarna lika får vi
x2 :
x1 :
x0 :
VL:s
HL:s täljares koefficienter
0 = A +C
⇔ A = −1, B = 2, C = 1
3 = A + B + 2C
1 = C
Alltså är
∫
(
)
x
2
3x + 1
2
g(x)dx =
= ln x−ln(x+1)−
+C = ln
−
+C .
2
x(x + 1)
x+1
x+1
x+1
T (x)
För f (x) = N(x)
med N(x) = ∏mj=1 (x−a j )n j , alla a j olika och n j ∈ Z+ och grad T <
grad N. Ansättningen vid PBU ser då ut så här:
f (x) = ∑
aj
Exempel 2.31
nj
A j,k
kj
k j =1 (x − a j )
∑
För att finna en primitiv funktion till
en logaritmisk och en arctan-term
(2.15)
2x + 1
x2 + 2x + 4
2x + 1
2(x + 1)
1
=
−
.
2
2
(x + 1) + 3 (x + 1) + 3 (x + 1)2 + 3
ser vi att vi kan få
2.7. GENERALISERAD INTEGRAL
33
Integralen av den första termen är ln((x + 1)2 + 3) + C1 . Integralen av den andra
termen:
(
)
∫
∫
1
1
1
1
x+1
dx =
dx = √ arctan √
+C2
( )2
(x + 1)2 + 3
3
3
3
x+1
√
+
1
3
Svar: En primitiv funktion är
(
)
( 2
)
1
x+1
ln x + 2x + 4 − √ arctan √
.
3
3
Beräkna integralen
Exempel 2.32
∫ 2
x+1
0
x2 + 4
dx.
Lösning: Integranden går att dela upp i partialbråk så här
x
1
x+1
= 2
+ 2
.
2
x +4 x +4 x +4
Om vi deriverar F1 (x) := ln(x2 + 4) får vi F1′ (x) =
1
x2 + 4
· 2x. Detta hjälper oss att
integrera den första termen. För den andra termen skriver vi
∫
1
1
dx =
2
x +4
4
∫ 2
x+1
0
x2 + 4
∫
[
dx =
dx
1
= · 2 arctan(x/2) +C .
2
(x/2) + 1 4
]1
1
1
· ln(x2 + 4) − · arctan(x/2) =
2
2
0
1
(π + ln 16)
8
2.7 Generaliserad integral
2.7.1
Definition och inledande exempel
Om integrationsintervallet inte är kompakt (slutet och begränsat) eller om integranden inte är begränsad i integrationsintervallet, kallar man integralen generalis∫ b
f (x)dx med a < b. Vi koncentrerar oss på två fall.
erad. Betrakta
a
I a = −∞ eller b = ∞, d.v.s. obegränsat integrationsintervall.
II f (x) → ±∞, då x → a+ eller x → b− .
Exempel 2.33
Beräkna integralen
∫ 4
dx
0
√ .
x
KAPITEL 2. INTEGRAL
34
Lösning:
Vi vet att f (x) := √1x → ∞, då x → 0+ , d.v.s.
integranden är obegränsad i integrationsintervallet [0, 4] . Vi löser likväl integralen med
primitiv funktion.
y
∫ 4
[√ ]4
√
x1/2
dx
√ = 2 x 0 = 2·2 = 4 .
= 2 x så att
F(x) =
1/2
x
0
Att integralen existerar som reellt tal innebär
att integralen är konvergent.
Exempel 2.34
y=
1
x
1
Beräkna integralen
∫ ∞
(3x + 1)e−x/2 dx .
0
Lösning: Man säger att denna integral "är generaliserad i oändligheten". Vi
beräknade en primitiv funktion i exempel 2.23. Vi beräknar integralen över ett
intervall [0, b] och låter sedan b → ∞.
∫ b
0
[
]b
(3x + 1)e−x/2 dx = −2e−x/2 (3x + 7) = 14 − 2(3b + 7)e−b/2 .
0
Nu låter vi b −→ ∞. Vi vet att exponentialfunktionen "vinner" över potensfunktionen 2(3b + 7), d.v.s. 2(3b + 7)e−b/2 −→ 0. Alltså är integralens värde 14 − 0 = 14
(och därmed konvergent).
∫ 1
Exempel 2.35
Vi beräknar integralen
ln x dx.
0
Lösning:
∫ 1
0
ln x dx = {P.I.} = [x ln x]10 −
∫ 1
0
1
x · dx = [x ln x − x]10 = 1 ln 1 − 1 − (0 ln 0 − 0) ,
x
men vad är 0 ln 0? Svar: Detta uttryck är inte definierat. I stället skall vi beräkna
integralen, för a > 0,
∫ 1
a
ln x dx = {P.I.} =
[x ln x]1a −
∫ 1
a
1
x · dx = [x ln x − x]1a = 1 ln 1 − 1 − (a ln a − a)
x
och sedan beräkna lim a ln a − a = 0 − 0 = 0, ett känt gränsvärde. Svar den gena→0+
eraliserade integralen är −1.
∫ ∞
Exempel 2.36
Beräkna den generaliserade integralen
i exempel 2.30.
g(x)dx, där g(x) är given
1
4
x
2.7. GENERALISERAD INTEGRAL
35
Lösning: Vi integrerar då över intervalltet [1, b] = {x : 1 ≤ x ≤ b} och låter sedan
b → ∞.
∫ b
1
g(x)dx = − ln(b + 1) + ln 2 +
2
2
−
+ ln b − ln 1 .
1+1 b+1
Vi sammanför termer av liknande typ innehållande b.
(
)
∫ b
2
b
1/b
+ ln
·
=
g(x)dx = 1 + ln 2 −
b+1
b + 1 1/b
1
(
)
1
2
= 1 + ln 2 −
+ ln
→ 1 + ln 2, då b → ∞ .
b+1
1 + 1/b
Svar: Den generaliserade integralen är konvergent med värdet 1 + ln 2 .
Exempel 2.37
Beräkna den generaliserade integralen
∫
dx
.
R cosh x
Lösning: cosh x =
∫ ∞
2 dx
x
−x
−∞ e + e
ex + e−x
, så att integralen kan skrivas
2
= {symmetri} =
∫
=
Exempel 2.38
∫ ∞
4dx
0
ex + e−x
=
(π π)
4ex dx
x b
=
4
lim
[arctan(e
)]
=
4
·
−
= π.
0
b→∞
e2x + 1
2 4
Beräkna den generaliserade integralen
∫
R
4dx
x4 + 4
.
Lösning: Integrationsintervallet är R = (−∞, ∞) och integralen betyder alltså
∫ ∞
4dx
−∞
x4 + 4
.
Integranden är en positiv rationell funktion. grad nämnare = 4 > grad täljare = 0
och 4−0 > 1, så att integralen är konvergent. Dessutom skall integranden utvecklas
med PBU. Först en faktoruppdelning av nämnaren, här m.h.a. kvadratkomplettering.
x4 + 4 = (x2 )2 + 2 · x2 · 2 + 22 − (2x)2 = (x2 + 2x + 2)(x2 − 2x + 2) .
Vi kan utnyttja att integranden är jämn ch integrationsintervallet symmetriskt och
får
∫ ∞
∫ ∞
4 dx
8 dx
=
.
4 +4
4 +4
x
x
−∞
0
KAPITEL 2. INTEGRAL
36
Alltså ansätter vi
8
x4 + 4
=
A · (2x + 2) + B C · (2x − 2)x + D
+
=
x2 + 2x + 2
x2 − 2x + 2
=
2Ax3 − 2Ax2 + 4A + Bx2 − 2Bx + 2B + 2Cx3 + 2Cx2 − 4C + Dx2 + 2Dx + 2D
(x2 − 2x + 2) (x2 + 2x + 2)
Vi identifierar nu koefficienterna i respektive leds täljare.
x3 :
x2 :
x1 :
x0 :
0
0
0
8
=
=
=
=
2A + 2C
x3 : −A = C
−2A + B + 2C + D
x2 : 4A = B + D = 2B
⇔ 1
⇔
−2B + 2D
x : B = D
4A + 2B − 4C + 2D
x0 : 8 = 8A + 4B = 16A
B = D=1
A = −C =
Integralen är alltså
[
]∞
∫ ∞
)
8 dx
1(
2
2
=
ln(x + 2x + 2) − ln(x − 2x + 2) + arctan(x + 1) + arctan(x − 1) =
4
2
0 x +4
0
( 2
)
]b
x + 2x + 2
1
ln 2
+ arctan(x + 1) + arctan(x − 1) =
= lim
b→∞ 2
x − 2x + 2
0
[
[
=
lim
b→∞
(
− ln
=
(
)
]
1
1 + 2/b + 2/b2
ln
+
arctan(b
+
1)
+
arctan(b
−
1)
−
2
1 − 2/b + 2/b2
)
02 + 2 · 0 + 2
− arctan(0 + 1) − arctan(0 − 1) =
02 − 2 · 0 + 2
1
π π 1
π π
ln 1 + + − ln 1 − + = π .
2
2 2 2
4 4
Kommentarer
• Man
∫ ∞ kan genom att utnyttja resultatet i föregående exempel, lätt beräkna
dx
.
4
−∞ x + 1
∫ ∞
Exempel 2.39
Betrakta den generaliserade integralen
0
dx
√
.
ex − 1
(a) På vilka sätt är integralen generaliserad?
(b) Beräkna integralen.
1
.
Lösning: Sätt integranden till f (x) := √ x
e −1
(a) Integralen är generaliserad dels genom att övre gräns är ∞ och dels för undre
gräns x = 0 gäller att f (x) → ∞, då x → 0+ .
(b) Vi neräknar nu integralen och försöker oss på V.S
t=
√
2t dt
= dx
ex − 1 ⇔ ln(t 2 + 1) = x ⇒ 2
t +1
1
2
2.7. GENERALISERAD INTEGRAL
37
med gränser
x − gränser 0 ∞
t − gränser
0 ∞
Integralen kan alltså skrivas
∫ ∞
1
0
2t dt
= 2[arctant]∞
0 = π.
t t2 + 1
·
Betrakta integralen
Exempel 2.40
∫ ∞
dx
√
.
x xα + 1
1
(a) För vilka reells α är integralen konvergent?
(b) Beräkna integralen för dessa α.
Lösning:
(a) Vi kan jämföra integranden på följande sätt.
1
1
√
≤ 1+α/2
α
x x +1 x
och integralen
∫ ∞
1
1
x1+α/2
dx
är konvergent omm α > 0. Således är integralen konvergent, om α > 0, enligt
Jämförelsekriteriet. För α = 0 får vi integranden
1
√
x· 2
som ger en divergent integral. För α ≤ 0 är alltså integralen divergent, återigen enligt Jämförelsekriteriet.
√
(b) Vi beräkna nu integralen för α > 0. Sätt t = xα + 1. Detta ger
x = (t 2 − 1)1/α ⇒ dx =
1 2
(t − 1)1/α−1 · 2t dt
α
och gränserna blir
x − gränser 1
t − gränser
∞
√
2 ∞
Integralen blir alltså
1
α
∫ ∞
1
1
· (t 2 − 1)1/α−1 · 2t dt =
√
1/α
2
α
·t
2 (t − 1)
[ (
)]
1
t −1 ∞
=
=
ln
√ 2
α
t + 1 √2
2 t −1
∫ ∞
2 dt
(√
)
(√
)
(
)
)
1
1 − 1/b
1
2−1
1
2+1
2 (√
lim ln
− ln √
= ln √
= ln
2+1
b→∞ α
1 + 1/b
α
α
α
2+1
2−1
KAPITEL 2. INTEGRAL
38
∫ ∞
x dx
=: I är icke-elementär. Det betyder att
ex − 1
en primitiv funktion inte kan uttryckas i elementära funktioner. Man kan ändå visa
att
∫ ∞
x dx
π2
=
.
x
6
0 e −1
∫ ∞
x dx
Vad är
=: J?
x
0 e +1
Exempel 2.41
Följande integral
0
Lösning: Vi skriver om integranden utom faktorn x, m.h.a. PBU på följande sätt.
1
ex − 1
=(
ex/2
1
)2
−1
=
A
ex/2 − 1
+
B
ex/2 + 1
.
Genomt aqtt föra liknämnigt och sedan sätta täljarna lika får vi
{
A = 1/2
⇔
B = −1/2
1 : 1 = A−B
ex/2 : 0 = A + B
Alltså är
I=
∫ ∞
x dx
0
ex − 1
=
1
2
∫ ∞
0
x dx
1
−
ex/2 − 1 2
∫ ∞
0
x dx
.
ex/2 + 1
I de två sista integralerna gör vi V.S. x/2 = t ⇔ dx = 2dt, som ger att
I=
∫ ∞
2t dt
0
et − 1
−
∫ ∞
2t dt
0
Alltså är
1
I=J=
2
et + 1
= 2I − 2J ⇔ 2J = I .
∫ ∞
xdx
0
ex + 1
=
π2
.
12
Exempel 2.42
Vi visar nu, i en integral, som övergår i en generaliserad integral,
vikten av att den nya variabeln är en funktion av den gamla (Se exempel 2.16 sidan
24).
Beräkna integralen
∫ 4π
dx
.
0 2 + sin x
Vi vet att V.S. t = tan(x/2) gör att integranden blir en rationell funktion i t, av inte
allt för hög grad. Men med denna V.S. får vi inte att x = x(t). Vi ser att t−gränserna
blir t = α = 0 och t = β = 0. Detta är orimligt eftersom integranden > 0. Vi kan
skriva integralen som
∫ 4π
0
dx
=2
2 + sin x
∫ 2π
0
dx
2 + sin x
eftersom integranden har perioden 2π. Vi vet att för t = tan(x/2) är x = x(t) =
2 arctant, (Se (2.13) sidan 26) alltså att x är en funktion av t för −∞ < t < ∞ med
2.7. GENERALISERAD INTEGRAL
39
intervallet −π < x < π. Återigen gör periodiciteteten för sin x att vi kan förskjuta
integrationsinvervallet från (0, 2π) till (−π, π). Integralen kan alltså skrivas
∫ 4π
0
dx
=2
2 + sin x
∫ ∞
2
∫ 2π
0
dx
=2
2 + sin x
∫ π
dx
= {t = tan(x/2)} =
−π 2 + sin x
1
2dt
=
2
2t
t +1
−∞
2+ 2
t +1
∫ ∞
=2
dt
= {K.K.} = 2
2
−∞ t + t + 1
∫ ∞
dt
4
= 2·
2
3
−∞ (t + 1/2) + 3/4
∫ ∞
dt
√
√
=
−∞ (2t/ 3 + 1/ 3)2 + 1
]∞
√
√
√
3
4 ( π π ) 4π
8
=√ .
·
arctan((2t/ 3 + 1/ 3)
=√
+
=
3 2
2 2
3
3
−∞
[
2.7.2
Generaliserad integral forts
∫ 1
dx
för olika α. För α = 1 fårvi en
xα
primitiv funktion ln x, så att då integralen divergent. För a > 0 är
[ 1−α ]1
∫ 1
∫ 1
)
dx
1 (
x
−α
1−α
=
=
x
dx
=
1
−
a
.
α
1−α a 1−α
a x
a
Exempel 2.43
Vi beräknar nu integralen
0
Omm 1 − α > 0, d.v.s. omm α < 1 är existerar gränsvärdet
)
1 (
1
.
1 − a1−α = ... =
a→0+ 1 − α
1−α
lim
Precis för dessa α är integralen alltså konvergent. För integralen
vi får divergens om α = 1. Vi beräknar sedan
∫ b
dx
1
xα
=
∫ ∞
dx
1
xα
, ser vi att
b1−α
1
−
.
α−1 α−1
Detta uttryck har ett (ändligt) gränsvärde, omm α > 1.
Vi kan sammanfatta exemplet ovan med att
(i)
∫ 1
dx
0
(ii)
xα
∫ ∞
dx
1
xα
är konvergent ⇔ α < 1
(2.16)
är konvergent ⇔ α > 1
∫ b
f (x)dx med a < b är konvergent eller
Ett bra sätt att avgöra om en integral
a
divergent är att beräkna integralen och på så sätt se vad man får när man låter den
x = b →"generaliserade" gränsen.
∫ 4
Exempel 2.44
Är integralen
0
dx
√ konvergent eller divergent?
x+ x
KAPITEL 2. INTEGRAL
40
Lösning: Vi kan substituera i denna integral, ex.vis x = t 2 för t ≥ 0. Då är dx =
√
2tdt och x = t. Gränserna är t = 0 och t = 2 .
∫ 4
0
dx
√ =
x+ x
∫ 2
2tdt
0
t2 + t
=2
∫ 2
dt
0
t +1
= [2 ln(t + 1)]20 = 2(ln 3 − ln 1) = 2 ln 3 ,
och alltså konvergent.
∫ ∞
Exempel 2.45
Är följande integral konvergent?
0
dx
√
.
x3 + 1
∫ 1
dx
√
existerar, eftersom integranden är kontinuerlig i
0
x3 + 1
det kompakta integrationsintervallet [0, 1]. För [1, ∞) gäller att
Lösning: Vi har att
1
1
1
≤√
= 3/2
0≤ √
3
3
x +1
x +0 x
och integralen
∫ ∞
dx
x3/2
integralen konvergent.
1
är konvergent (med α = 3/2 > 1 i (ii).) Alltså är den givna
1
1
I det sista exemplet jämför vi med integranden √
≥ 0 med 3/2 , som är
3
x
x +1
större och vars generaliserade integral är konvergent. Då är också den ursprungliga
integralen konvergent, enligt jämförelsekriteriet.
2.8. VOLYMBERÄKNING
41
2.8 Volymberäkning
2.8.1
Volym av vissa kroppar
Exempel 2.46
(Skivmetoden) Vi skall beräkna volymen av en rak cirkulär kon
med radie r och höjd h. Vi låter ytan som begränsas av y = f (x) = kx, y = 0
(x−axeln), samt x = 0 och x = h rotera kring x−axeln. Detta genererar en rotationskropp.
Dess volym fås om man "skivar"kroppen i, säg n tunna skivor med tjocklek ∆x =
h−0
i punkterna x = i ∆x med med radie y = k (i ∆x) för i = 0, 1, 2, ..., n−1. Vidare
n
r
är linjens riktningskoefficient k = .
h
y
r
h
x
Vi får då en undersumma (och p.s.s. en översumma) och till stlut låta n → ∞, och
därmed få volymen av konen. Istället skall vi mur resonera med infinitesimala,
dV och dx, d.v.s. volymen dV av en oändligt liten cylinderskiva med oändligt
liten tjocklek dx. Man hoppar därmed över steget med under- och översumma och
får direkt en integral för volymen, här betecknad V . En sådan cylinderskiva har
volymen dV := πy2 dx. Konens volym är integralen ("summan" av alla dV )
∫ h
V
=
∫ h
dV =
x=0
= π
0
πy2 dx = π
( r )2 h3 πr2 h
· =
.
h
3
3
∫ h ( )2
r
0
h
· x2 dx = π
( r )2 [ x3 ]h
h
3
0
=
KAPITEL 2. INTEGRAL
42
2.9 Volym och Areaberäkning
2.9.1
Volym, skalmetoden
Exempel 2.47
Man kan dela in konen i koncentriska cylindriska skal parallela med
x−axeln.
r
Ett sådan skal har den infinitesimala arean dy (h − x)2πy, där y = kx = x. Vi skall
h
alltså integrera m.a.p. y, där 0 ≤ y ≤ r. Integralen blir
)
∫
) 1
h r(
h
y h − y dy = 2π
ry − y2 = πhr2 ,
r
t 0
3
∫ r (
2π
0
d.v.s. samma resultat som i exempel 29.
2.9.2
Area av yta
Vi tar återigen konen som exempel.
Exempel 2.48
y
r
ds
x
h
x
Ett infinitesimalt ytelement är har den infinitesimala arean 2πyds, där ds =
Vi får att konens mantelyta är
∫ h
∫ h
A=
2πyds =
x=0
r
= 2π ·
h
2.9.3
∫
√
2
2
2πy dx + dy = 2π
x=0
√
1+
( r )2 ∫
h
0
0
h
r
x dx = 2π ·
h
√
1+
√
h
rx
h
(
dy
1+
dx
dx2 + dy2 .
)2
dx =
( r )2 h2
√
· = πr · h2 + r2 .
h
2
Kurva och kurvlängd
Exempel 2.49
√
Beräkna längden av kurvan γ: x 7→ (x, ln x), där 1 ≤ x ≤ e.
2.9. VOLYM OCH AREABERÄKNING
Lösning: Kurvans längd är
√
∫ e
L = |γ| =
∫ e
ds =
x=1
1
(
dy
1+
dx
43
)2
∫ e√
1 + (1/x)2 dx =
dx =
1
 √
x2 √
+1 = t



√

∫ e

x
=
t2 − 1
x2 + 1
tdt
=
dx =
dx = √

x
1


t2 −

√
√1

x = 1,t = 2 =: ax = e,t = e2 + 1 =: b
= {pol.div} =
∫ b 2
t −1+1
a
t2 − 1
∫ b(
dt =
a
1
1+ 2
t −1











)
dt = {PBU} =
[
])
)]
[
(
1
1
1
1
t −1 b
1+
−
dt = t + ln
=
2 t −1 t +1
2
t +1 a
∫ b(
=
a
(
)
(
)
1
b−1
1
a−1
= (b − a) + ln
− ln
=
2
b+1
2
a+1
=
=
( (√
)
(√
))
e2 + 1 − 1
2−1
ln √
− ln √
= {Visa:}
2+1
e2 + 1 + 1
√
√
1
e2 + 1 − 2 +
2
√
√
√
√
e2 + 1 − 2 + ln( e2 + 1 − 1) + ln( 2 + 1) − 1 = 2.00...
∫ b
t2
a
t2 − 1
=
dt =
KAPITEL 2. INTEGRAL
44
2.10 Tyngdpunkt, tröghetsmoment m.m.
2.10.1
Tyngdpunkt
Exempel 2.50
Givet två punktmassor m1 och m2 utplacerade på en x−axel med
koordinater x1 respektive x2 . Bestäm tyndgpunktens x−koordinat xT .
Lösning:
m1
m2
x1
xT
x2
Tyngdpunktens koordinat uppfyller
(xT − x1 )m1 = (x2 − xT )m2
elller ekvivalent
m1 x1 + m2 x2
= xT ,
m1 + m2
alltså ett viktat medelvärde av x1 och x2 . Om man p.s.s. sätt har en massa m
n
∑ ∆mi , i koordinaterna xi , i = 1, 1, 2..., n
uppdelad i n delmassor ∆mi , d.v.s. m =
i=1
blir tyngdpunktens koordinat
n
∑ xi ∆mi
xT =
i=1
m
.
För en massa som är "kontinuerligt" utbreddd längts x−axeln är tyngdpunktens
x−koordinat (Obs! Summasymbolen ersätts av ett integraltecken och ∆mi av dm
etc.)
∫
1
x dm .
xT =
m
Exempel 2.51
Bestäm koordinaterna för tyngdpunkten för konen med konstant
densitet ρ given i exempel 29.
Lösning: Tyngdpunktens y−koordinat är 0, av symmetriskäl. Dess massa är m =
πr2 h
ρV , där volymen V =
. Vi skall se volymen som en funktion av x. V (x) =
3
2
πy x
, där V (h) = V , d.v.s. volymen av (del-)konen där höjden är x och 0 ≤ x ≤ h.
3
r
Eftersom y = x, så är
h
( r )2 πx3
och m(x) = ρ ·V (x) .
V (x) =
h
3
Vi låter K ∫representera kroppens, d.v.s. hela konens koordinater. Vi skall lösa
integralen xdm. Vi byter variabel från m till x.
K
dm = ρ
( r )2
h
· x2 dx .
2.10. TYNGDPUNKT, TRÖGHETSMOMENT M.M.
45
Vi skall alltså lösa integralen
∫
∫ h
xdm =
K
0
( r )2 ∫
( r )2 h4 ρπr2 h2
ρπ
· x3 dx = ρπ
=
.
h
h
4
4
Men detta är täljaren! Tyngdpunktens x−koordinat är
xT =
3
ρπr2 h2 3
1 ρπr2 h2
·
=
·
= h.
m
4
ρπr2 h
4
4
Tyngdpukten är alltså 1/4 från konens bas.
Exempel 2.52
För en roterande kropp finns en rotationsaxel. Dess vinkelhastighet
(eller vinkelfrekvens) ω definieras som vinkel per tidsenhet. Jorden roterar kring
sin polaxel med ett varv, d.v.s. 2π på tiden t = 24 h. Alltså är
ω=
2π
π
π
=
rad/h =
rad/s .
24 h 12
43200
Vi kan formellt definiera
ω=
dθ
dt
(2.17)
där t är tid och θ är vinkeln i radianer.
Göteborg ligger ungefär på breddgraden α = 57◦ . Avståndet till polaxeln är då
jordens radie r = 6400 km gånger cosinus för vinkeln. Alltså r cos α. Hastigheten
vpå denna breddgrad på jordytan är då
π
· 6.4 · 103 · cos 57◦ ≈ 913 km/h.
v = ω r cos α =
12
Exempel 2.53
Rörelseenergin för translationsrörelse är
mv2
(2.18)
2
där m är kroppens massa och v dess hastighet. Vi skall beräkna rörelseenergin Wk =
Wk,r för ett homogent klot med radie r, som roterar kring en axel (y−axeln) genom
dess medelpunkt. Vi antar att dess vinkelhastighet är ω. Sambandet mellan en
punkts hastighet v och vinkelhastigheten är v = ωx, där x är punktens (vinkelräta)
avstånd till medelpunkten (rotationsaxeln).
Wk,t =
y
r
x
x
KAPITEL 2. INTEGRAL
46
Vi tänker oss y−axeln som rotationsaxel med (konstant) vinkelhastighet ω. Punkter
på cylindern med avstånd x från rotationsaxeln (y−axeln) roterar med hastigheten
√
ω x = v. Volymen av cylinderskalet med radie x, tjocklek dx och höjd y = 2 r2 − x2
är då
√
dV = 2 r2 − x2 · |{z}
2π x · |{z}
dx .
| {z }
omkrets tjocklek
höjd
Motsvarande infinitesimala rörelseenergi är därmed
√
1
1
1
dWk = dm · v2 = ρ dV (ωx)2 = ρ xdx r2 − x2 ω2 x2 .
2
2
2
Den totala rörelseenergin är då
∫
Wk =
K
1
2
dWk =
= 2πω2 ρ
∫ r
∫ r
x=0
x3
√
0
√
ρ · 2π · x · 2 r2 − x2 ω2 x2 dx =
r2 − x2 dx .
Vi beräknar integralen.
∫ r
x
3
√
0
∫ r
=
2
r2 − x2 dx
r x
0
√
=
∫ r( √
x
0
r2 − x2 dx −
∫ r
0
r2 − x2 (x2 − r2 ) + r2 x
√
r 2 − x2
)
dx =
r2 x(r2 − x2 )3/2 dx =
[
)3/2 r2 ( 2
)5/2
r2 ( 2
=
r − x2
−
r − x2
3
5
]0
=
r
r5 r5 2r5
− =
.
3
5
15
Därmed är
Wk,r = 2πω2 ρ ·
4πr3
4πr3 r2 2
r2
2r5
= {Vklot =
} = ρ·
· ω = m · ω2 .
15
3
3
5
5
Detta skriver man om så att det liknar rörelseenergi för translationsrörelse, givet i
(2.18).
1 2mr2 2
1
Wk,r =
ω =: I ω2 .
2 5
2
2mr2
Uttrycket I :=
kallas kroppens (klotets) tröghetsmoment1 m.a.p. en axel
5
(y−axeln) genom klotets medelpunkt.
Exempel 2.54
Hur mycket rörelseenergi Wk,r har jorden? Antag att jorden har
konstant densitet.
Lösning:
• Jordens massa m = 6 · 1024 kg.
• Vinkelhastgiheten är ω =
• Radien är r = 6.4 · 106 m.
1 Moment
of intertia, I
2π
/s.
24 · 3600
2.10. TYNGDPUNKT, TRÖGHETSMOMENT M.M.
• Tröghetsmomentet är I =
2mr2
.
5
• Alltså är rörelseenergin
Wk,r =
1
· I · m · ω2 ≈ 1.6 · 1029 Joule .
2
47
KAPITEL 2. INTEGRAL
48
2.11 Lite om kurvor i planet
2.11.1
Parametrisering av en kurva
Exempel 2.55
Vi beskriver en tågresa med konstant hastighet över Västgötaslätten)
som (x, y) tågets positition (i något koordinatsystem) vid tiden t (h), som (x, y) =
(x0 , y0 )+t(α, β). Vid tiden t = 0 befinner sig tåget på stationen i GBG och vid tiden
t = 1.5 (h) i Falköping. Tågets √
hastighet är (α, β) km/h, ex.vis (α, β) = (60, 80)
(km/h). Dess fart är |(α, β)| = 602 + 802 = 100 km/h. Kurvan (som här är en
linje) är parametriserad med parametern t (som här är tid i h).
Exempel 2.56
Kurvan y = x2 har x som parameter.
Kurvan (x(t), y(t)) = (2t,t 2 − 1) kan ritas genom att sätta ut några punkter. Lättare
är dock att eliminera parametern. Eftersom t = x/2 kan vi skriva y = (x/2)2 − 1 =
x2
− 1, alltså en parabel.
4
En cirkel kan inte beskrivas som en funktion y av variabeln x. Däremot kan den
bekrivas m.h.a. en parameter. Ex.vis cirkeln med medelpunkt i (1, −2) och radie 3
kan skrivas
(x, y) = (3 cost + 1, 3 sint − 2), 0 ≤ t < 2π .
Exempel 2.57
Givet kurvan (x, y) = ( f (t), g(t)) = (cos3 t, sin3 t),
0 ≤ t ≤ 2π.
a) Bestäm tangentens ekvation där t = π/3.
b) Skissa kurvan.
c) Beräkna kurvans längd.
d) Beräkna den area som den innestängda ytan har.
Lösning:
a)
√
g′ (t)
3 sin2 t cost
=
= − tant ⇒ k(π/3) = − 3 .
′
2
f (t) −3 sint cos t
(
√ )
1 3 3
Linjen går genom punkten (x0 , y0 ) = ( f (π/3), g(π/3)) =
,
.
8 8
Linjens ekvation är då
k(t) =
y=−
b)
1√
3(2x − 1)
2
2.11. LITE OM KURVOR I PLANET
49
y
1
1
x
c) Kurvans längd är
∫ π/2 √
L = 4
f ′ (t)2 + g′ (t)2 dt = 4
∫ π/2
(9 sin2 t cos4 t + 9 sin4 t cos2 t)1/2 dt =
0
0
∫ π/2 √
= 4
9 sin
2
t cos2 t dt
∫ π/2
=3
0
sin 2tdt = 6 .
0
d)
∫ 1
A = 4
∫ 1(
1 − x2/3
ydx = 4
0
0
= {x = sin3 t} = 4
∫ π/2
= 4
∫ π/2
0
)3/2
d
(1 − sin2 t)3/2 3 sin2 t cost dt =
3 cos4 t sin2 t dt .
0
Nu är denna integrals värde densamma om man byter roller på sin och
cos, Alltså är
∫ π/2
A = 6
(cos4 t sin2 t + cos2 t sin4 t)dt = 6
0
=
3
2
∫ π/2
0
∫ π/2
(sint cost)2 dt =
0
3
sin 2t dt =
4
2
∫ π/2
0
(1 − cos 4t)dt =
3π
.
8
Exempel 2.58
Ex. vis för ellipsen med medelpunkt i (x, y) = (0, 0) och halvaxlar
a > 0 och b > 0 längs x− respektive y−axeln, kan man parametrisera ellipsens
ekvation genom


x = a cost
0 ≤ t ≤ 2π


y = b sint
Ellipsens längd L ges då av
√
∫ 2π ( )2 ( )2 ∫ 2π √
dg
df
+
=
L=
a2 sin2 t + b2 cos2 t dt .
dt
dt
0
0
50
KAPITEL 2. INTEGRAL
Detta är en integral, som inte kan lösas med primitiv funktion uttryckt i elementära
funktioner, utom i fallet a = b. I detta fall är kurvan en cirkel. Man kan alltså inte
få ett enkelt uttryck för ellipsens längd. Integralen kallas elliptisk integral.
Kapitel 3
Differentialekvation
3.1 Differentialekvation (DE)
3.1.1
Linjär DE med konstanta koefficienter av ordning 1
Exempel 3.1
För ekvationen y′ = 2x måste y = x2 + C. Vi kan rita y för olika
konstanter C.
y
2
y=x2+1
1
x
Vi säger att y = x2 +C är lösningen på DE:n.
Vi kan faktiskt ställa ett (rand- eller begynnelse-)villkor på den funktion som uppfyller DE:n, nämligen att kurvan (x, y(x)) skall gå genom en specifik punkt, i detta
fall ex.vis genom (x, y) = (−1, 2). Detta kan skrivas y(−1) = 2, som alltså är
2 = y(−1) = (−1)2 +C = 1 +C ⇔ C = 1 så att y = x2 + 1 .
Detta villkor ger alltså ett specifikt värde på konstanten C.
Exempel 3.2
Betrakta DE:n y′ (x) = −2y(x). Den är av ordning 1, ty y′ är den
högsta derivatan. kan skrivas y′ + 2y = 0 (där vi undertrycker den oberoende variabeln.) Den är linjär p.g.a. termerna är y′ och 2y. En linjär DE kategoriseras
vidare.
• Den är homogen ty HL= 0.
51
KAPITEL 3. DIFFERENTIALEKVATION
52
• Den är har konstanta koefficienter 1 och 2.
dy
dy
= −2y ⇔
= −2dx. Vi har därmed separerat varidx
y
ablerna. DE:n är separabel.
• DE:n kan skrivas
Vi kan lösa DE:n (med variablerna separerade), genom att integrera.
dy
= −2dx ⇔
y
∫
dy
= −2
y
∫
dx ⇔
ln y = C − 2x ⇔ y = eC−2x = eC · e−2x = C1 e−2x .
Exempel 3.3
DE:n y′ · y + 1/x = 0, y(1) = 2 är inte linjär, p.g.a. termen y′ · y. Den
måste vi lösa med variabelseparation.
dx
y ·y = −1/x ⇔ y dy = − ⇔
x
′
∫
y dy = −
∫
√ √
dx
y2
⇔ = C −ln x ⇔ y = ± 2 C − ln x .
x
2
Randvillkoret y(1) = 2 ger att det endast är "+" som gäller d.v.s.
√ √
√ √
y = + 2 C − ln x, y(1) = 2 = 2 C − ln 1 ⇒ C = 2 .
√
Alltså är y = 4 − 2 ln x
Exempel 3.4
DE:n xy′ + 2y = −2x är inhomogen, pg.a. HL och har icke-konstanta
koefficienter (x framför y′ ). Den går inte att lösa med ovanstående metoder. I stället
löses den med integrerande faktor, IF.
1. Dividera med x så att koefficienten framför y′ blir 1, så att man får
2
y′ + y = −2.
x
2
2. Beteckna koefficienten framför y med f = f (x) = .
x
3. Bestäm en primitiv funktion F(x) = 2 ln x till f (x).
( )2
4. Den integrerande faktorn I.F. är eF(x) = e2 ln x = eln x = x2 .
2
5. Multiplicera DE:n y′ + y = −2 med I.F. Detta ger
x
x2 y′ + 2xy = −2x2 . och VL = (x2 y)′ samt HL = −2x2 .
6. Integrera båda led:
x2 y = −
Exempel 3.5
∫
2x2 dx = C −
Lös DE:n y′ (t) + 2y(t) = 4t 2
C 2x
2x3
⇔y= 2 − .
3
x
3
3.1. DIFFERENTIALEKVATION (DE)
53
Lösning: Vi har redan löst y′ (t) + 2y(t) = 0 (Exempel 3.2), som är en homogen
DE. Den lösningen är y = Ce−2t . Man kallar denna lösningen för homogenlösningen yh till DE:n y′ (t) + 2y(t) = 4t 2 . Den kommer att ingå som en term i lösningen
till denna DE. Den andra termen svarar mot det speciella HL 4t 2 och kallas partikulärlösning (= y p ). Detta ansätter man med ett "matchande" uttryck. Eftersom
HL är ett polynom av grad 2 bör y p = At 2 + Bt +C. Insatt i DE:ns VL får vi
2At 2 + 2At + 2Bt + B + 2C = 4t 2 .
Genom att identifiera koefficienterna med varandra, får vi
B = −2,
A = 2,
C = 1 d.v.s. y p = 2t 2 − 2t + 1 .
Lösningen på DE:n är
y = yh + y p = Ce−2t + 2t 2 − 2t + 1 .
Kommentarer
• DE:n y′ (t) + 2y(t) = 0 är linjär, homogen med konstanta koefficienter. Den
har en lösning på formen y = Cert , där någon konstant r. Genom att sätta in
detta y i DE:n får vi
y′ (t) + 2y(t) = Crert + 2Cert = Cert (r + 2) = 0 .
Om inte C = 0, så måste r + 2 = 0, som är DE:ns karakteristiska ekvation,
med rot r = −2. Alltså är y = Ce−2t .
• Man kan också lösa DE:n i exemepel 3.5 med I.F. Då får man att
(
)′
e · y = 4t e ⇔ e · y =
2t
2 2t
2t
∫
4t 2 e2t dtx = {P.I.} = (2t 2 − 2t + 1)e2t +C
En tillämpning, fritt fall
Exempel 3.6
En fallande kropp med massa m, i vårt gravitationsfält påverkas
av dels gravitationskraften mg =: F1 och luftmotståndet F2 . Vi antar att den har
hastigheten 0 vid tiden t = 0. Vi antar att detta är proportionellt mot hastigheten v.
D.v.s. F2 = −k v för en proportionalitetskonstant k > 0 med positiv referensriktning
nedåt.
KAPITEL 3. DIFFERENTIALEKVATION
54
F2=-kv
Kropp med massa m
F1=mg
Den totala kraften F som påverkar kroppen är då F1 −F2 = F = {enl. Newtons kraftekvation} =
ma, där a är kroppens acceleration. Vi får alltså
F1 − F2 = mg − kv = ma ⇔ ma + kv = mg .
Eftersom a = v′ = v′ (t) är detta en linjär DE , inhomogen av första ordningen med
konstanta koefficienter, som kan skrivas
mv′ + kv = mg,
v(0) = 0 .
Den kan faktiskt lösas med variabelseperation (P.g.a. att HL är en konstant, d.v.s.
oberoende av tiden t).
dv
mv′ + kv = mg ⇔ m = mg − kv ⇔ dt =
dt
mdv
m
mdv
=−
⇔ t +C = − ln |kv − mg| ⇔
mg − kv
kv − mg
k
−
k
t +C1 = ln |kv − mg| = ln(mg − kv) eftersom v(0) = 0.
m
Vi tar e upphöjt till båda led.
eC1 e−kt/m = C2 e−kt/m = mg − kv ⇔ v =
mg −C2 e−kt/m
.
k
Nu är v(0) = 0. Detta ger att
mg −C2
mg
⇔ C2 = mg så att v =
· (1 − e−kt/m ) .
k
k
Vissa nödvändiga egenskaper för denna lösning
Vi ser speciellt att v(0) = 0, vilket stämmer med begynnelsevillkoret.
Vad händer då t blir stort, d.v.s. då t → ∞? Då får vi gränsvärdet(gränshastigheten)
mg
mg
lim v =
. Alltså, i praktiken en sluthastighet som därmed är v1 :=
. Den
t→∞
k
k
beror på kroppens massa och planeten jordens tyngdacceleration, samt på "luftmotståndskoefficienten" k. Ju större den är desto lägre sluthastighet.
Accelerationen är (per definition)
dv
d mg
a(t) ≡ v′ (t) =
= {i detta fall} =
· (1 − e−kt/m ) =
dt
dt k
(
)
mg
k −kt/m
=
(0 − − · e
) = ge−kt/m .
k
m
0=
3.1. DIFFERENTIALEKVATION (DE)
55
Speciellt är v′ (0) = a(0) = g, d.v.s. vid tiden t = 0 har man accelerationen g, vilket
är naturligt.
Vad blir accelerationen a(t), då t ökar? Vi låter t → ∞. Då får vi gränsvärdet a = 0,
d.v.s. kroppens acceleration går mot noll. I praktiken uppnår kroppen ganska snart
en konstant hastighet och accelerationen blir (därmed) 0.
En differentialekvation av ordning n i en funktion y,
dny
är en ekvation som innehåller n :e derivatan y(n) (x) = n , som högsta
dx
derivata.
Definition 3.1
3.1.2
Olika typer av DE
• y′ (t) = −2y(t) kan ekvivalent skrivas y′ (t) + 2y(t) = 0. P.g.a. att termerna är
av typ A y′ och B y, så kallas DE:n linjär.
P.g.a. att HL= 0 kallas den homogen.
Eftersom A = 1 och B = 2, konstanter, så har den konstanta koefficienter.
Denna typ löses med karakteristisk ekvation.
y′
+x = 0 är inte linjär (Icke-linjär). Denna typ löses med variabelsepy2
aration.
• DE:n
• DE:n y′ (x) = −xy(x) kan skrivas y′ + xy = 0 och är alltså linjär och homogen
• DE:n y′ (x) = −xy(x) + x kan skrivas y′ + xy = x och är linjär och inhomogen
ty HL x ̸= 0 med icke-konstanta koefficenter.
• DE:n y′′ (t) + π2 y(t) = 0 är linjär homogen med konstanta koefficienter och
av ordning 2.
Samtliga DE ovan utom den sista är av första ordningen eftersom den högsta ordningen av derivata är 1.
Exempel 3.7
Lös DE:n y′ · y + x = 0.
Lösning: Vi skriver DE:n så här med åtföljande omskrivningar som separerar
variablerna x och y.
... ⇐⇒
dy
= −x ⇐⇒ y dy = −x dx
dx
Nu är variablerna x och y separerade. Därefter följer integration av båda led.
∫
y dy = −
∫
√
x dx ⇔ y2 = −x2 +C ⇔ y = ± C − x2 .
Exempel 3.8
a) Lös DE:n y′ (x) = −xy(x).
KAPITEL 3. DIFFERENTIALEKVATION
56
Lösning:
Denna DE kan lösas med variabelseparation. Vi får
x2
x2
x2
dy
x2
= −xdx ⇔ ln y = C − ⇔ y = eC− 2 = eC · e− 2 = C1 e− 2
y
2
b) Lös DE:n y′ (x) = −xy(x) + x.
Lösning:
Vi kan skriva DE:n som
y′ + xy = x .
Den är linjär, med icke-konstanta koefficienter (x framför y) och inhomogen,
p.g.a. x i HL. Den är vidare av 1:a ordningen. En sådan DE löses med
integrerande faktor (IF). Man ser till att faktorn framför y′ är 1, vilket är
fallet här. Funktionen framför y är x =: f (x). Man tar sedan en primitiv
x2
funktion till denna F(x) = . Den integrerande faktorn är
2
I.F. = eF(x) = ex
2 /2
.
Man multiplicerar sedan DE:n med den integrerande faktorn.
= ex
2 /2
(y′ + xy) = ex
2 /2
x.
2
VL är då en produkt av I.F. ex /2 och den sökta funktionen y. D.v.s. Vi får
ekvationen och följande ekvivalenta ekvationer
∫
( 2
)′
2
2
x2 /2
x2 /2
x /2
· y = ex /2 xdx = ex /2 +C ⇔
e
·y = e x ⇔ e
y = 1 +Ce−x
2 /2
(svar) .
3.2 De av ordning 2
Vi tar bar upp linjära DE av andra ordningen.
3.2.1
Linjär DE med konstanta koefficienter av ordning 2
Exempel 3.9
DE:n y′′ − 4y = 0 är en linjär, homogen, DE av andra ordningen med
konstanta koefficienter. Vi utgår från att lösningen är termer på formen y = Cerx ,
(om nu y är en funktion av variabeln x). Karakteristisk ekvation är r2 − 4 = 0 med
rötter r = ±2. Lösningen till DE:n är y = C1 e2x +C2 e−2x .
Exempel 3.10
DE:n y′′ +4y = 0 är linjär och homogen med konstanta koefficienter.
Karakteristisk ekvation är r2 + 4 = 0 ⇔ r = ±2i. Så att
y = C1 e2ix +C2 e−2ix = C1 (cos 2x + i sin 2x) +C2 (cos 2x − i sin 2x) =
= (C1 +C2 ) cos 2x + i(C2 −C2 ) sin 2x = A cos 2x + B sin 2x .
3.2. DE AV ORDNING 2
57
Exempel 3.11
y
y=0
F
En fjäder är belastad med en vikt (kula) med massa m. Kulna och fjädern sätts i
svängning i lodrät ledd kring jämviktsläget y = 0. Krafterna som påverkar fjädern
är −k · y, där k > 0 är fjäderkonstanten. Newtons kraftekvation ger ma = −ky ⇔
2
m · ddt 2y + k · y = 0, en linjär DE av ordning 2 med konstanta koefficienter. Den
är dessutom homogen (Inga utifrån verkande krafter). Denna DE har lösningen
k
y = A cos ωt + B sin ωt, där ω2 = . Med begynnelsevillkoret y(0) = 0, d.v.s.
m
utslaget 0 vid tiden t = 0 blir A = 0. så att
y = B sin ωt .
Exempel 3.12
Antag att y = y(t) är en funktion som uppfyller DE:n
y′′ (t) + 2y′ (t) + 5y(t) = 0, y(0) = 0, y′ (0) = 2 .
Bestäm funktionen y!
Lösning: DE:n är av ordning 2, linjär homogen och med konstanta koefficienter.
Dess karakteristiska ekvation, med lösning, är
r2 + 2r + 5 = 0 ⇐⇒ r = −1 ± 2i .
Vi får lösningen
y = C1 e(−1+2i)t +C2 e(−1−2i)t = {cos(−t) = cost, sin(−t) = − sint} =
= e−t (C1 (cos 2t + i sin 2t) +C2 (cos 2t − i sin 2t)) =
= e−t (A cos 2t + B sin 2t) .
KAPITEL 3. DIFFERENTIALEKVATION
58
Rand- eller begynnelsevillkoren ger att
{
y(0) = 0 = A och
y′ (0) = 2 = −A + B
så att y = 2e−2t sint (Svar).
3.2.2
Inhomogen Linjär DE av andra ordningen
Exempel 3.13 De:n y′′ +4y = 8e2x har en lösning y som kan delas upp i en homogenoch en partikulärlösning. yh = A cos 2x + B sin 2x och y p ansätter vi som y p = Ce2x .
Då blir y′′p = 4Ce2x . Insatt i DE:n ger detta
y′′p + 4y p = 4Ce2x + 4Ce2x = 8Ce2x = 8e2x ⇔ C = 1 .
Lösningen på DE:n är därmed
y = yh + y p = A cos 2x + B sin 2x + e2x .
Exempel 3.14
Lös DE:n

′′
−t

y (t) + 4y(t) = 5e + 6 sin(t),


y(0) = 5, y′ (0) = −1
Lösning: yh = A cos 2t + B sin 2t. Vi har två olika typer av termer i HL, så vi tera
fram två olika partikulärlösningar.
y p1 = Ae−t ,
och y p2 = B cost +C sint .
Dessa sättes in en och en i DE:n med motsvarande HL.
y′′p1 (t) + 4y p1 (t) = Ae−t + 4Ae−t = 5e−t ⇒ A = 1 .
y′′p2 (t)+4y p2 (t) = −B cost −C sint +4(B cost +C sint) = 6 sint ⇔ A = 0,
B = 2.
Alltså är y(t) = e−t + 2 sint + A cos 2t + B sin 2t. Vi har nu två begynnelsevillkor
(eller randvillkor), som är
5 = y(0) = 1 + A ⇔ A = 4
och
−1 = y′ (0) = −1 + 2 − 2A · 0 + 2B · 1 ⇔ −2 = 2B ⇔ B = −1 .
Svar: y(t) = e−t + 2 sint + 4 cos 2t − sin 2t .
3.2. DE AV ORDNING 2
59
Kommentarer
• Observera att konstanterna A, B och C används i olika betydelser.
• Vi ansätter alltså y p2 = B cost + C sint, även om det bara är en sin−term i
HL.
Exempel 3.15
y
y=0
F
En fjäder är belastad med en vikt (kula) med massa m. Kulan och fjädern sätts i
gungning i lodrät ledd kring jämviktsläget y = 0 (som i exempel 3.11. Krafterna
som påverkar fjädern är −k · y, där k > 0 är fjäderkonstanten. Newtons kraftekva2
tion ger ma = −ky ⇔ m · ddt 2y + k · y = 0, en linjär DE av ordning 2 med konstanta
koefficienter.
(a) Vi kan tänka oss att fjädern+kulan utsätts av en yttre kraft F1 = e−t , d.v.s.
my′′ + ky = e−t .
Funktionen y, som lösning till denna DE, består av två termer yh = A cos ωt +
k
B sin ωt, där ω2 =
och y p = Ce−t och y p insatt i DE:n ger
m
my′′p + ky = (m + k)Ce−t = e−t ⇔ C =
så att
y = yh + y p = A cos ωt + B sin ωt +
1
m+k
e−t
.
m+k
(b) Nu tänker oss att vi har en yttre kraft 4m sin ωt, d.v.s. av samma typ som en
term i homogenlösningen. DE:n är
my′′ (t) + ky(t) = 4m sin ωt
och med
k
= ω2 kan den skrivas
m
y′′ +
k
y?y′′ + ω2 y = 4 sin ωt .
m
KAPITEL 3. DIFFERENTIALEKVATION
60
Att ansätta y p = C1 cos ωt +C2 sin ωt ger inget eftersom dessa termer ingår i
yh . I stället skall vi ta till en komplex metod med z = z(t) = x(t) + i y(t) =
x + i y och HL 4eiωt . Då är Im z = y lösningen på vår DE. Vi får DE:n
z′′ + ω2 z = 4eiωt ,
sätt z = u · eiωt .
Denna DE blir då
(
)
eitω u′′ (t) + 2iωu′ (t) = 4eiωt .
Dels blir det en DE av ordning 2 men utan nolltegradsterm och dels kan ei ωt
förkortas. Denna DE kan lösas, först genom att sätta u′ = v varvid vi får en
DE av ordning 1.
ic1 e−2itω
2it
u(t) =
+ c2 −
.
2ω
ω
z(t) = u(t)eiωt =
c1 sin(ωt)
ic1 cos(ωt)
+ ic2 sin(ωt) +
+
2ω
2ω
+ c2 cos(ωt) +
2t sin(ωt) 2it cos(ωt)
−
,
ω
ω
var imaginärdel är
y = y(t) = A sin ωt + B cos ωt −
2t cos(ωt)
,
ω
2t cos(ωt)
som är DE:ns lösning. Observera att lösningen innehåller termen −
,
ω
som är en obegränsad funktion i t. Detta betyder att fjäder och vikt svänger
allt högre amplitud, då t ökar.
Exempel 3.16
Givet DE:n
y′′ + 3y′ + 2y = 4e−x .
Lös DE:n!
Lösning: DE:n är av andra ordningen, linjär inhomgeon med konstanta koefficienter (1, 3 och 2). Vi delar upp y = yh + y p Vi börjar med yh . Karakteristisk
ekvation är
r2 + 3r + 2 = 0 ⇐⇒ r = r1 = −1, r = r2 = −2, som ger yh = Ae−x + Be−2x .
Nu till y p . Vi ser att HL ingår som term i yh . Då duger inte ansättningen y p = C e−x .
I stället (knep 2), sätt y = e−x · u, där u = u(x) är en ny funktion. då blir
y′ = e−x u′ − e−x u = e−x (u′ − u)
och
y′′ = e−x u′′ − 2e−x u′ + e−x u
Insatt i DE:n vi
y′′ + 3y′ + 2y = e−x (u′′ + u′ ) = 4e−x ⇐⇒ u′′ + u′ = 4 ⇐⇒ u′ + u = 4x +C1 .
3.2. DE AV ORDNING 2
61
Den sista DE:n kan vi lösa med I.F.= ex . Multiplikation med denna faktor i den
sista DE:n ger
(ex u)′ = (4x +C1 )ex ⇐⇒ ex u = (4x +C1 )ex −
∫
4ex dx = (4x +C2 )ex +C3 .
Nu är
y = e−x u = e−2x ((4x +C2 )ex +C3 ) = (4x +C2 )e−x +C3 e−2x .
Kommentarer
• Givet en linjär DE
n
∑ ak · y(k) = g(x)
(3.1)
k=0
där an ̸= 0 och alla koefficienter ak konstanta.
Då, kan man visa, kan VL skrivas som ett polynom i D "gånger" y.
(
)
n
∑ a k · Dk
y = p(D)y
(3.2)
k=0
Med bytet y = eα x u kan detta VL skrivas
n
eα x · ∑ bk u(k) .
(3.3)
k=0
För att veta hur koefficienterna ak och bk hänger ihop, måste man skriva
dy
y′ =
= D y. Då är
dx
y′ = D y = eα x (D + α)u
och allmänt
y(k) = Dk y = eα x (D + α)k u, k = 0, 1, 2, 3, ...
(3.4)
• För en DE, som y′′ + 3y′ + 2y = 4e−x kan VL skrivas
(D2 + 3D + 2)y = (D + 2)(D + 1)y .
Med bytet y = eαx u kan man lätt övertyga sig om att DE:n:s VL blir
eαx (D + 2 + α)(D + 1 + α)u .
I detta exempel väljer vi α = −1 och får
e−x (D + 1)D u = {Sätt lika med HL} = 4e−x .
Vi ser att DE:ns VL inte innehåller någon term med u, utan endast u′ och u′′
och att vi kan förkorta e−x och får en enklare DE.
• RÅDET är att, för en linjär DE med konstanta koefficienter och ett HL
A er x som ingår som term i yh , sätt
y = er x u
• Sambandet mellan (3.2) och (3.3) är
p(D)y = eαx p(D + α)u .
Exempel 3.17
KAPITEL 3. DIFFERENTIALEKVATION
62
Reflexion av lodrät parallellt infallande strålar på en spegel
Hur skall en spegel vara formad för att parallellt infallande strålar skall reflekteras
mot ett gemensamt fokus F? Detta är ett klassiskt problem löst redan under Antiken. Vi skall här komma fram till lösningen via en differentialekvation.
infallande stråle
y
normal
y= yHxL
F
r=i i
Αr
x
Α
tangent
I figuren ser vi att
α = i och i den gröna triangeln får vi vinkelsumman α + π/2 + i + π − αr = π (=
180◦ ). Eftersom i = α, får vi
2α + π/2 = αr .
Nu kan vi uttrycka tan αr i tan α. Observera att tan αr är riktningskoefficienten för
den reflekterade linjen. Och denna linje går genom punkterna (0, F) och skärningspunkten med kurvan och alltså genom (x, y). Denn linjes riktningskoefficent är
y−F
y−F
=
. Vi får alltså att
alltså
x−0
x
tan αr = tan(2α + π/2) =
y−F
och tan α = y′ .
x
M.h.a. trigonometri skall vi uttrycka tan(2α + π/2) i tan α:
sin(2α + π/2)
sin(2α + π/2)
cos(2α)
=
=
=
cos(2α + π/2) cos(2α + π/2) − sin(2α)
tan(2α + π/2) =
=
cos2 α − sin2 α 1/ cos2 α 1 − tan2 α
·
=
=
−2 sin α cos α 1/ cos2 α
−2 tan α
=
y′2 − 1
.
2y′
Detta ger nu differentialekvationen
y′2 − 1 y − F
y−F
=
⇔ y′ =
±
′
2y
x
x
√(
y−F
x
)2
+1.
y−F
= z ⇔ y−F = x z ⇒ y′ = xz′ +z. Insatt i ekvationen
x
ovan, där vi bortser från "minus" i "±", får vi
Här behvös ett knep. Sätt
xz′ + z = z +
√
z2 + 1 ⇔
dx
dz
=√
⇔
x
z2 + 1
∫
dx
=
x
∫
dz
√
.
z2 + 1
3.2. DE AV ORDNING 2
63
Vi löser dessa integraler.
√
√
ln x +C = ln(z + z2 + 1) ⇔ kx = z + z2 + 1 ⇔
(kx − z)2 = k2 x2 − 2kxz + z2 = z2 + 1
⇐⇒
y−F
k x = 2kx ·
+ 1 = 2k(y − F) + 1 .
x
2 2
Vi sätter in randvillkoret y(0) = 0, som ger
0 = 2k(0 − F) + 1 ⇐⇒ k =
1
2F
så att ekvationen ovan blir
1
1
y
x2
2
·
x
=
(y
−
F)
+
1
=
⇐⇒
y
=
.
4F 2
F
F
4F
64
KAPITEL 3. DIFFERENTIALEKVATION
Kapitel 4
Taylorutvecklingar
4.1 Taylorpolynom m.m.
4.1.1
Fakultet
0! = 1, (läses "noll-fakultet"). 1! = 1. Vidare är 2! = 1 · 2 = 2 och 3! = 1 · 2 · 3 = 6.
Allmänt för n = 1, 1, 2, ..., n! = 1 · 2 · 2... · (n − 1) · n. Ex.vis är 52! ≈ 1067 antalet
(k − 1)! 1
= .
ordningar som korten i en kortlek med 52 kort kan ligga. Speciellt är
k!
k
4.1.2
Några funktioners kurvor och dito Taylorpolynom
Exempel 4.1
• Vi ritar funktionen y = f (x) = ex och y = 1 + x för x ≈ 0.
2.5 y y=f HxL=ex
2.0
1.5
y=x+1
1.0
0.5
-1.0
0.5
-0.5
x
1.0
• Nedan: Kurvan y = f (x) = ex och kurvan y = 1 + x + x2 /2 för x ≈ 0
65
KAPITEL 4. TAYLORUTVECKLINGAR
66
7
6
y= f HxL=ex
5
4
3
2
y=1+x+
x2
2
1
-2
4.1.3
1
-1
2
Hur man bestämmer ett sådant polynom
Exempel 4.2
I den sista figuren kallar vi polynomet p2 (x). Vi ser att f (x) och p2 (x)
sammanfaller i x = 0, d.v.s. f (0) = p2 (0). Likaså är f ′ (0) = p′2 (0). I detta exempel
är x = 0 den punkt som vi identifierar funktion och polynom. Polynomet är ett
Maclaurinpolynom (utveckling kring x = 0), ett specialfall av ett Taylorpolynom
(utveckling kring x0 .) Vi utvecklar
∫ x
f (x)
=
f0 (x) +
x0
[
]x ∫ x
f ′ (t)dt = {P.I.} = f0 (x) + (t − x) f ′ (t) x − (t − x) f ′′ (t)dt =
0
[
x0
]x ∫ x
(t − x)2 ′′
(t − x)2 ′′
f (t) +
f (t)dt =
2
2
x0
x0
=
{P.I.} = f0 (x) + (x − x0 ) f ′ (x0 ) −
=
f0 (x) + (x − x0 ) f ′ (x0 ) +
(x − x0 )2 ′′
f (x0 ) +
2
=
f0 (x) + (x − x0 ) f ′ (x0 ) +
[
]x ∫ x
(x − x0 )2 ′′
(t − x)3
(t − x)3 (4)
−
f (x0 ) +
f (t)dt =
2
2 · 3 x0
2·3
x0
=
f0 (x) + (x − x0 ) f ′ (x0 ) +
(x − x0 )2 ′′
(x − x0 )3 ′′′
f (x0 ) +
f (x0 ) −
2
2·3
∫ x
(t − x)2 ′′
f (t)dt = {P.I.} =
2
x0
∫ x
(t − x)3 (4)
f (t)dt .
x0
2·3
Vi får för en funktion f (x) som är n + 1 ggr kontinuerligt deriverbar att
f (x) =
+
f (x0 ) ·
f
(n)
(x − x0 )1
(x − x0 )2
(x − x0 )
+ f ′ (x0 ) ·
+ f ′′ (x0 ) ·
+ ...+
0!
1!
2!
(x − x0 )n
(x0 ) ·
+ (−1)n
n!
= pn (x) + Rn (x) .
∫ x
(t − x)n (n+1)
f
(t)dt =
x0
n!
4.1. TAYLORPOLYNOM M.M.
67
Exempel 4.3
Vi skall bestämma Maclaurinpolynomet för f (x) = ex av grad n.
f (0) = f ′ (0) = f ′′ (0) = ... = f (n) (0) = 1. Alltså är Maclaurinpolynomet
ex ≈ 1 + x +
x2 x3
xn
+ + .... + .
2! 3!
n!
Man kan få en bra numerisk uppskattning av e. Sätt x = 1 och n = 2. Detta ger
x2
och p2 (1) = 2.5. P.s.s. är p4 (1) ≈ 2.70833, p6 (1) ≈ 2.71806.
p2 (x) = 1 + x +
2
De första 100 siffrorna i decimalutvecklingen av e är
e = 2.718281828459045235360287471352662497757247093699959574966
967627724076630353547594571382178525166427...
Vi har att
ex = 1 + x +
∞ k
x2
xn
x
+ ... + + ... = ∑ .
2!
n!
k=0 k!
(4.1)
Vi byter x mot ix i (4.1). Vi får då
Exempel 4.4
(
)
x 2 x4
x3 x5
e = cos x + i sin x = 1 − + + − ... + i x − + − ...
2! 4!
3! 5!
ix
Detta ger alltså Maclaurinserierna för cos x och sin x (Identifiera realdelarna med
varandra och p.s.s. med imaginärdelarna.)
Exempel 4.5
a) Bestäm Taylorpolynomet av grad 1, 2 och 3 av f (x) = 2x3 − 2x + 3 i x =
x0 = 1.
Lösning:
f (x) = 2x3 − 2x + 3,
f ′ (x) = 6x2 − 2,
f ′′ (x) = 12x,
f (3) (x) = 12 ,
så att
f (1) = 3,
f ′ (1) = 4,
f ′′ (1) = 12,
f (3) (1) = 12 .
p0 (x) = 3
p1 (x) = p0 (x) + 4(x − 1) = −1 + 4x
(x − 1)2
= −1 + 4x + 12x2 /2 − 12x + 6 = 5 − 8x + 6x2
p2 (x) = p1 (x) + 12 ·
2!
p3 (x) = 3 − 2x + 2x3
b) Bestäm Maclaurinpolynomet av e2x · cos x av grad 3.
KAPITEL 4. TAYLORUTVECKLINGAR
68
Det gäller att utveckla de två faktorerna tillräckligt långt.
Lösning:
e2x = 1 + 2x +
(2x)2 (2x)3
x2 x4
+
+ ... och cos x = 1 − + + ...
2!
3!
2! 4!
Detta ger
e2x cos x = (1+2x+2x2 +4x3 /3+...)(1−x2 /2+x4 /24+...) = 1+2x+
Svar: 1 + 2x +
∫ 1
c) Beräkna
av grad 3.
3x2 x3
+ +...
2
3
3x2 x3
+ .
2
3
esin x dx, genom att integerera motsvarande Maclaurinpolynom
0
Lösning:
esin x = 1 + (sin x) +
(sin x)2 (sin x)3
+
+ ... ≈
2
6
≈ 1 + (x − x3 /6) +
≈ 1+x+
)2 1 (
)3
1(
x − x3 /6 + x − x3 /6 + ... =
2
6
x2 x4
− =⇒
2
6
∫
1
x2
5
+x+1 ⇒
p3 (x)dx = ... = ≈ 1.67 .
2
3
0
En numerisk beräkning ger 1.63187.
p3 (x) =
4.2 Fler Maclaurinserier
Exempel 4.6
Den geometriska serien är
a + ax + ax2 + ax3 + ... = a(1 + x + x2 + x3 + ...) = a ·
1
, om |x| < 1 . (4.2)
1−x
Vi kan modifiera den. Först sätter vi a = 1 och sedan byter vi x mot −x och får
1 − x + x2 − x3 + ... =
1
,
1+x
|x| < 1 .
Om vi integrerar denna serie på intervallet [0, x] (och samtidigt byter x mot t som
integrationsvariabel), och om termvis integration är tillåten, får vi
ln(x + 1) = x −
∞
xk
x2 x3
− + ... = ∑ (−1)k−1 ,
2
3
k
k=1
som är Maclaurinserien för ln(x + 1). Vi skall senare visa att den är konvergent för
|x| < 1. P.s.s. kan vi byta x mot −x2 i (4.2) och få
∞
1
=
(−x2 )k ,
∑
1 + x2 k=0
4.2. FLER MACLAURINSERIER
69
som vi integerar på intervallet [0, x]. Detta ger
∞
arctan x =
x2k+1
∑ (−1)k 2k + 1 .
k=0
Man kan visa att HL är konvergent om |x| < 1.
Exempel 4.7
a) Beräkna gränsvärdet
x sin x
arctan2 x
Man kan lösa gränsvärdet ovan m.h.a. Taylorutveckling, eller med Taylorserie. I
detta fall beräknar vi serien i x0 = 0, d.v.s. en Maclaurinserie. Täljaren är
lim
x→0
x sin x = x(x − x3 /3! + x5 /5! + ...)
och nämnaren är
(
)2
x3 x5
x − + + ...
3
5
så att
f (x) x(x − x3 /3! + x5 /5! + ... 1/x2 1 + termer av högre grad
1+0
= (
=
→
= 1.
)2 ·
2
3
5
g(x)
1/x
1
+
termer
av
högre
grad
1+0
x
x
x − 3 + 5 + ...
b) Beräkna gränsvärdet
(x sin x)2
x→0 x2 − tan2 x
lim
Lösning: Vi skriver om täljaren som
T (x) := x2 (x − x3 /3! + x5 /5! − ...)2 = x4 − x6 /3 + 2x8 /45 + termer av högre grad
Nämnaren skriver vi som
N(x) = (x − tan x)(x + tan x)
och
′′′
f (x) := tan x
f ′ (x) = 1 + tan2 x
f ′′ (x) = 2 tan x(1 + tan2 x)
f (x) = 2(1 + tan2 x + 3 tan2 x(1 + tan2 x))
f (0) = 0
f ′ (0) = 1
f ′′ (0) = 0
f (0) = 2
så att f (x) = tan x = x +
′′′
2 3
x + termer av högre grad. Därmed är
3!
N(x) = (x − (x + x3 /3 + termer av högre grad))(x + (x + x3 /3 + termer av högre grad)) =
= −
2x4
+ termer av högre grad .
3
Därmed är
(x sin x)2
x4 + termer av högre grad 1/x4
1 + termer av högre grad
=
=
·
4
2
2
4
2x
x − tan x − + termer av högre grad 1/x
−2/3 + termer av högre grad
3
och det är klart att det sista uttrycket går mot
3
1
= − då x → 0.
−2/3
2
KAPITEL 4. TAYLORUTVECKLINGAR
70
4.2.1
Lite knep för att bestämma Taylorutveckling m.m.
1. För funktionen f (x) = cos2 x, kan man utnyttja omskrivningen f (x) =
och sedan utnyttja att
cos 2x = 1 −
1 + cos 2x
2
(2x)2 (2x)4
+
− ...
2!
4!
Ex.vis är Maclaurinpolynomet av cos2 x av grad 3 lika med
(
)
(2x)2
1 1
+
1−
= 1 − x2 .
2 2
2!
2. För den sammansatta funktionen ecos x kan Maclaurinpolynomet av grad 4
tas fram genom att se att x = 0 ger cos x = cos 0 = 1. Därför skall den yttre
funktionen ez med z = cos x utvecklas i z = 1. Vi vill dock utnyttja att vi
känner Maclaurinpolynomen för ez . Därför skriver vi om funktionen som
ecos x = ecos x−1+1 = e · ecos x−1
och nu är den inre funktionen z = cos x − 1, som är noll om x = 0. Därför
kan vi utveckla ecos x−1 = ez i x0 = 0. Den inre funktionen z = cos x − 1 =
x2 x4
− + − ... Fortsättningen blir
2! 4!
ecos x = e · ecos x−1 = e · ez = e(1 + z + z2 /2! + z3 /3! + ...) =
( 2
)
( 2
)2
( 2
)3
x
x4
1
x
x4
x
x4
1
= e(1 + − + − ... +
− + − ... +
− + − + ...) =
2! 4!
2
2! 4!
6
2! 4!
[
]
)
(
)
(
1
x 2 x4
x2
4 1
+
+ ... = e 1 − + + ...
= e 1− +x
2
24 8
2
6
)
(
x2 x4
cos
x
Maclaurinpolynomet av grad 4 av funktionen e
är alltså e 1 − +
2
6
(Svar).
3. Man brukar säga att allt som liknar ett Taylorpolynom också är ett Taylorpolynom.
4.3 Fyra satser
Vi antar att f (x) är en kontinuerlig funktion i intervallet [a, b], a ≤ b.
Teorem 4. 1 (Satsen om största och minsta värde) f (x) antar ett största och ett
minsta värde på [a, b], säg fmax och fmin .
Teorem 4. 2 (Satsen om mellanliggande värde) f (x) antar alla värden mellan det
största och minsta värdet.
Annorlunda uttryckt; För varje y: fmin ≤ y ≤ fmax , finns ett x−värde ξ i intervalelt
[a, b], sådant att f (ξ) = y.
Teorem 4. 3 (Triangelolikheten för integral)
Med samma förutsättningar som ovan är
∫ b
∫ b
| f (x)| dx .
f (x) dx ≤
a
a
(4.3)
4.4. OMSKRIVNING AV RESTTERMEN
71
Bevis:
Eftersom a ≤ b och ± f (x) ≤ | f (x)|, så är
∫ b
a
± f (x)dx = ±
∫ b
a
f (x)dx ≤
∫ b
a
| f (x)| dx .
Dett är ekvivalent med (4.3).
Teorem 4. 4 (En medelvärdessats för integraler) Antag dessutom att g(x) är kontinuerlig i [a, b] och att g(x) inte växlar tecken i intervallet. Då finns ξ ∈ [a, b],
sådant att
∫
∫
b
b
f (x) g(x) dx = f (ξ)
a
g(x) dx .
(4.4)
a
Bevis:
Vi antar att g(x) ≥ 0 och att
∫ b
g(x) dx =: B > 0. Då är
A
fmin g(x) ≤ f (x) g(x) ≤ fmax g(x)
där fmin och fmax är definierade ovan. Vi integrerar alla tre uttrycken och får då
bibehållna olikheter, d.v.s.
fmin · B ≤
∫ b
a
f (x) g(x) dx ≤ fmax · B
Detta kan ekvivalent skrivas
∫b
fmin ≤
a
f (x) g(x) dx
=: y ≤ fmax .
B
Enligt satsen om melenliggande värde, finns ξ ∈ [a, b], sådan att
∫b
a
f (ξ) = y =
f (x) g(x) dx
.
B
Genom att multiplicera med B följer (4.4).
4.4 Omskrivning av resttermen
4.4.1
Lagranges form
Rn (x) = (−1)n
∫ x
(t − x)n (n+1)
f
(t) dt
x0
n!
(4.5)
(t − x)n
inte ändrar tecken för x0 ≤ t ≤ x eller x ≤ t ≤ x0 .
n!
Vi antar först att x0 ≤ t ≤ x. Då är t − x ≤ 0 för alla t ∈ [x0 , x]. Fallet x ≤ t ≤ x0
visas p.s.s. Alltså kan resttermen, m.h.a. (4.4) skrivas
Man kan lätt konstatera att
n
Rn (x) = (−1) f
Exempel 4.8
(n+1)
(ξ)
∫ x
(t − x)n
x0
n!
dt = (−1)n f (n+1) (ξ) = f (n+1) (ξ)
Bestäm de 4 första siffrorna i talet e. Utgå från att e ≤ 3.
(x − x0 )n+1
(n + 1)!
KAPITEL 4. TAYLORUTVECKLINGAR
72
Lösning: Vi använder Maclaurinutvecklingen av f (x) = ex . Resttermen Rn (x) =
eξ xn+1
. Vi sätter x = 1 och får
(n + 1)!
n
e=
1
∑ k! + Rn (1) .
k=0
Vi behöver ha n så stort att |Rn (1)| ≤ 10−4 . Vi ser snart att n = 7 ger att
|e − p7 (1)| = |R7 (1)| ≤
3
< 10−4 .
8!
Alltså räckerdet att bestämma de fyra första siffrorna i p7 (1):
p7 (1) = 2.71825...
så at e:s första fyra siffror är 2.718 .
4.4.2
Ordoform
Vi betecknar en begränsad funktion med B(x) (med olika index). För ex.vis gränsvärdesberäkning, då x → x0 , behöver man endast veta storleken på resttermen. Vi ser att
1
resttermen Bestär av två faktorer, dels f (n+1) (ξ) ·
, som är en begränsad
(n + 1)!
funktion B(x) av x, då x nära x0 .
Ett uttryck h(x) som kan skrivas B(x) · (x − x0 )k , där B(x) är begränsad för x nära x0 , betecknas O ((x − x0 )k ) .
Definition 4.1
Exempel 4.9
Ex.vis är
O (x − x0 )2 + O ((x − x0 )3 )
O (x − x0 )2 · O ((x − x0 )3 )
O (x − x0 )2 + (x − x0 )3
O (x − x0 )2 · (x − x0 )3
O (x − x0 )3
(x − x0 )2
Exempel 4.10
=
=
=
=
O ((x − x0 )2 )
O ((x − x0 )5 )
O ((x − x0 )2 )
O ((x − x0 )5 )
= O (x − x0 )
Beräkna gränsvärdet
(x sin x)2
.
x→0 x2 − tan2 x
lim
genom att använda ordoform.
Lösning: Vi låter x0 = 0, d.v.s. använder Maclaurinutvecklingar. Nämnaren:
2
x2 − (x + x3 /3 + O (x5 ))2 = x2 − x2 − x4 + O (x6 ) .
3
Täljaren:
x2 (x − x3 /3! + O (x5 ))2 = x4 + O (x5 )
4.5. MACLAURINUTVECKLING AV SINUS OCH COSINUS
73
och slutligen
Täljaren
x4 + O (x5 )
1/x4
1 + O (x2 )
3
= 2 4
·
=
→−
4
2
6
Nämnaren − 3 x + O (x ) 1/x
−2/3 + O (x )
2
då x → 0.
4.5 Maclaurinutveckling av sinus och cosinus
4.5.1
Sinus
Vi bestämmer den allmänna Maclaurinutvecklingen av sin x i x0 = 0. Vi behöver
d
d
komma ihåg att
cost = − sint och
sint = cost samt formeln för partiell indt
dt
tegration.
sin x
sin x − sin 0 =
=
∫ x
[
(t − x)2
x+
sint
2
=
1 · cost dt = {P.I.} =
0
[
]x
−
−
0
cost dt = x − (0 − 0) −
∫ x
(t − x)3
sint dt = x −
3!
0
∫ x
x3
−
3!
(t − x) sint dt =
∫ x
(t − x)2
0
∫ x
(t − x)3
3!
0
2
cost dt =
sint dt =
[
]x ∫ x
x3
(t − x)4
(t − x)4
−
sint +
cost dt =
3!
4!
4!
0
0
x3
x− −0+0+
3!
=
]x
0
2
0
0
(t − x)3
= {P.I.} = x −
cost
3!
= {P.I.} = x −
∫ x
(t − x)2
[(t − x) cost]x0 +
∫ x
(t − x)4
4!
0
cost dt .
x3
. Det betyder att vi
3!
har likhet mellan dito resttermer, d.v.s. R3 (x) = R4 (x). Man väljer alltid resttermen
med jämnt index, alltså R2 (x), R4 (x), R6 (x) etc. Därmed är
Vi ser att i de två sista raderna är polynomet detsamma, x −
n
sin x =
∑ (−1)k−1 ·
k=1
4.5.2 Cosinus
∫ x
Först iakttar vi att
0
vi
cos x = 1 −
∫ x
0
x2k−1
+ (−1)n
(2k − 1)!
0
(2n)!
1 · sint dt = {P.I.} = 1 − [(t − x) · sint]x0 +
(t − x)2
= 1−0+0+
cost
2!
x2
+
2!
x2
= 1− −
2!
cost dt .
(4.6)
(− sint) dt = cos x − cos 0, så att med partiell integration får
[
= 1−
∫ x
(t − x)2n
∫ x
(t − x)2
0
2!
∫ x
(t − x)3
0
3!
]x
+
0
∫ x
(t − x)2
0
2!
0
(t − x) cost dt =
sint dt =
sint dt = {P.I.} = 1 −
cost dt .
∫ x
[
]x ∫ x
x2
(t − x)3
(t − x)3
+
sint −
cost dt =
2!
3!
3!
0
0
KAPITEL 4. TAYLORUTVECKLINGAR
74
Även här får vi samma polynom med restterm R2 (x) och R3 (x), som alltså måste
vara lika. Vi kan nu skriva upp en allmän formel/identitet för cosinus och använder
resttermen med udda index.
n
cos x =
∑ (−1)k
k=0
x2k
+ (−1)n
(2k)!
∫ x
(t − x)2n+1
0
(2n + 1)!
cost dt .
(4.7)
Kapitel 5
Talföljd och serie
5.1 Talföljd och serie, inledande exempel
Vi har redan behandlat serier i samband med Taylorutveckling. Men låt oss börja
med följder. En följd är (per definition) en lista
(ak )∞
k=1 = (a1 , a2 , ..., ak , ...)
Exempel 5.1
a) (2k − 1)∞
k=1 är följden av alla udda positiva heltal. Utskrivet är detta
(2k − 1)∞
k=1 = (1, 3, 5, 7, ..., 2k − 1, ...) .
b)
( )∞
(1, 4, 9, 16, 25, ...) = k2 k=1
är följden av alla heltalskvadrater.
c) Oftast har vi följder eller serier, där det finns någon typ av system i följdens
element (eller seriens termer). Finns det något system/mönster i följande
följd?
1, 2, 3, 8, 10, 12, 13, ...
(
)
2n − 1 ∞
2
c) Följden
har ett gränsvärde. Ett element kan skrivas an = −
3n
3
n=1
1
2
och vi ser att an → . Vidare är följden växande; an ≤ an+1 , n =
3n
3
2
1, 2, 3, .... Den är dessutom uppåt begränsad av just . En följd med dessa
3
två egenskaper har "alltid" ett gränsvärde och är därmed konvergent.
Vi visar nu två gränsvärden för två följder.
(i) Om |x| < 1, så är lim xn = 0.
n→∞
Bevis: Vi logaritmerar |x|n och får n ln |x| < 0 eftersom |x| < 1. Således har
vi att n ln |x| → −∞, då n → ∞. Nu är
−|x|n ≤ xn ≤ |x|n
och både första och sista ledet → 0 då n → 0. Alltså följer det att lim xn = 0.
n→∞
75
KAPITEL 5. TALFÖLJD OCH SERIE
76
xn
= 0.
n→∞ n!
Bevis: För fixt x finns sett n0 , sådant att x < n för alla n ≥ n0 . Därmed är
x
< 1 för dessas n. Vi kan skriva
n
(ii) lim
( )n−n0 +1
|x|
|x|
|x|
x
|x|n |x| |x|
|x| |x| |x|
=
· · ... ·
· · ... ·
≤
· · ... ·
·
n!
1 2
n0 − 1 n0
n
1 2
n0 − 1
n0
(
och faktorn
x
n0
)n−n0 +1
→ 0, då n → ∞ enligt (i).
Exempel 5.2
För den geometriska följden gäller (per definition) att kvoten mellan
två konsekutiva element är konstant.
a0 = b,
a2 = bx2 , ..., ak = bxk , ...
a1 = bx,
ak+1
bxk+1
=
= x för alla k = 0, 1, 2, 3, .... Detta använder vi längre fram i kvot
ak
bxk
och rotkriterierna.
Exempel 5.3
∞
1 + 1/2 + 1/3 + 1/4 + ... + 1/k + ... =
1
∑k
k=1
kallas den harmoniska serien. En series summa skall upppfattas som ett (eventuellt)
gränsvärde, i detta fall
∞
n
1
1
:=
lim
∑ k n→∞ ∑ k .
k=1
k=1
Vi skall argumentera för att denna series summa är ∞, d.v.s. att serien är divergent.
y
y= f HxL=1x
1
1
Vi ser att
n
1
∑k≥
k=1
då n → ∞. Detta visar att
n
1
2
∫ n
dx
1
x
= ln n → ∞
∑ k → ∞, då n → ∞.
k=1
x
5.1. TALFÖLJD OCH SERIE, INLEDANDE EXEMPEL
77
Kommentarer
1
• Vi kan se termerna i serien ovan som ak = = f (k) för funktionen x 7→
k
1
f (x) = .
x
• Detta sätt att jämföra en icke-negativ serie (d.v.s. med termer ≥ 0) med en
generaliserad integral, kallas integralkriteriet. Mer exakt säger detta kriteriet
att om f (x) ≥ 0 för x ≥ 1 och f (x) är avtagande, så gäller att
∫ ∞
1
∞
f (x)dx konvergent ⇐⇒
∑ f (k) konvergent .
(5.1)
k=1
• Vi kan p.s.s. jämföra två serier vars element är an ≥ bn ≥ 0 för n = 1, 2, 3, ....
∞
Om
∑ bn divergerar, så divergerar serien mot ∞.
Och i så fall divergerar
n=1
∞
också
∑ an (också mot ∞).
n=1
• Ett nödvändigt villkor för konvergens. Antag att
∞
∑ ak är konvergent, d.v.s.
k=1
n
∑ ak = A
n→∞
lim
k=1
existerar som reellt (entydigt) tal A: Vi oberverar att
n
n−1
k=1
k=1
∑ ak − ∑ ak = an → A − A = 0
då n → ∞, d.v.s. termen an måste gå mot 0, då n → ∞. För den harmoniska
serien är termerna an = 1/n → 0 men serien är ändå divergent.
Exempel 5.4
Integralkriteriet (5.1) ger ex.vis att
∞
1
∑ k2
k=1
π2
är konvergent. Man kan t.o.m. visa att seriens summa är . Vi skall m.h.a. detta
6
resultat beräkna summan av serierna
T :=
∞
∞
1
(−1)k+1
och
U
:=
∑ (2k − 1)2
∑ k2 .
k=1
k=1
S :=
∑ k2 = 12 + 22 + 32 + 42 + ... =
∞
1
1
1
1
1
k=1
1
1
1
1
1
1
+ 2 + 2 + ... + 2 + 2 + 2 + ... =
2
1
3
5
2
4
6
1
3
π2
T + · S =⇒ T = · S =
.
4
4
8
P.s.s. skriver vi om
U=
1
1
1
1
1
3
1
π2
−
+
−
+
...
=
T
−
S
=
S
−
S
=
.
12 22 32 42
4
4
4
12
KAPITEL 5. TALFÖLJD OCH SERIE
78
Integralkriteriet ger att
∞
1
∑ kα är konvergent
⇐⇒ α > 1 .
(5.2)
k=1
∞
Den geometriska serien
Exempel 5.5
1
∑ xk = 1 − x , omm |x| < 1. För andra x är
k=0
den divergent.
Om 0 ≤ ak ≤ bk och
∞
∞
∑ bk
är konvergent, så verkar det klart att även
∑ ak
är
k=1
k=1
konvergent.
Nedan följer ett exempel på hur två serier med positiva termer kan jämföras, för
att visa konvergens.
∞
Ett jämförelsekriterium: Serien
Exempel 5.6
1
∑ (2k − 1)√k
har termen ak =
k=1
1
√
√ . En sådan term har en nämnare som är samma storleksordning som
2k k − k
1
1
för termen bk = √ = 3/2 . För att visa att dessa två termer är av samma storlekk k k
sordning, tar vi gränsvärdet av kvoten
ak
1
1
1
k
=
→
= < ∞ då k → ∞ .
=
bk 2k − 1 2 − 1/k
2−0 2
Vi sluter oss till att 0 ≤ ak ≤ (1/2 + δ)bk , för något δ > 0 och eftersom gränsvärdet
1
är < ∞, så är även
2
∞
∑ ak =
k=1
Alltså
∞
är
∞
∞
∞
1
1
√
≤
(1/2
+
δ)
b
=
(1/2
+
δ)
k
∑ (2k − 1) k
∑
∑ k√k < ∞ .
k=1
k=1
k=1
1
∑ (2k − 1)√k konvergent.
k=1
Vi har tidigare betraktat geometrisk följd och serie. Att den geometriska serien
ak+1 bxk+1
=
= x, är till sitt belopp < 1.
konvergerar är ekvivalent med att kvoten
ak
bxk
Detta leder till kvotkriteriet för konvergens för en serie med termer ak ≥ 0.
5.1.1 Kvotkriteriet
ak+1
= ρ och ρ < 1, så är den
k→∞ ak
konvergent. Detta betyder ju att ak+1 ≈ ak · ρ för alla tillräckligt stora k,
d.v.s. för k ≥ k0 för något k0 .
Kvotkriteriet Om, för en icke-negativ series termer lim
∞
∑ ak ≈ ak ρk−k
0
k=k0
0
5.1. TALFÖLJD OCH SERIE, INLEDANDE EXEMPEL
79
som är en konvergent geometrisk serie. Mer exakt bevisar man konvergensen
så här. För tilltäckligt stora k, d.v.s. det finns ett k0 , sådant att för k ≥ k0 , så
är 0 ≤ ak ≤ ρ + ε, för ett ε > 0 sådant att ρ + ε =: x < 1. Därmed är summan
∞
∞
k=k0
k=k0
∑ ak ≤ ∑ x k
och den högra serien är konvergent.
Rottestet följer längre fram.
Med ak = b · xk och x = 2−1/12 är serien
Exempel 5.7
∞
∑ ak konvergent. Summan
k=0
2b
=: L och är längden på gitarrsträngen på en gitarr om bandens bredder
2 − 211/12
är bxk , k = 0, 1, 2, ....
är
b
Band på gitarrhals
b x b x2
Out[57]=
I
II
III
IV
XII
L2
L
Med L = 63.0 cm blir b = 3.53592 cm.
Exempel 5.8
∞
a)
Avgör om följande serier är konvergenta eller divergenta.
2
∑ k(k + 1) .
k=1
1
Lösning: Vi kan jämföra seriens termer ak med bk = 2 , som ger en konk
vergent serie.
ak k + 1
=
= 1/2 + 1/(2k) → 1/2
bk
2k
som ger att serien är konvergent. Vi kan t.o.m. räkna ut seriens summa.
ak =
2
1
1
= −
k(k + 1) k k + 1
och
n
∑ ak
k=1
=
1 1 1 1 1 1 1 1
1
− + − + − + − + ... −
=
1 2 2 3 3 4 4 5
n+1
= 2−
2
−→ 2, då n −→ ∞ .
n+1
En sådan summa/serie kallas teleskopsumma/serie.
KAPITEL 5. TALFÖLJD OCH SERIE
80
( √ )k
√
3
3
√
√ <1
b) ∑
. Denna serie är en geometrisk serie med |x| =
1+ 3
k=1 1 + 3
och alltså konvergent. Även denna series summa går att beräkna.
∞
( √ )k
( √ )k−1
√
√
3
3 ∞
3
3
√
=
∑ 1 + √3 = 1 + √3 ∑ 1 + √3
1+ 3
k=1
k=1
∞
( √ )j
3
∑ 1 + √3 =
j=0
∞
√
√
√
3
1
3
√
√ ·
√
√ = 3.
=
1 + 3 1 − √3
1+ 3− 3
1+ 3
5.1.2
Rotkriteriet
Jämte kvotkriteriet finns rotkriteriet. Dessa är tillämpligt på ungefär samma typ
∞
av serier. Om vi utgår från den geometriska serien
∑ axk , så tar vi k−roten ur
k=0
|bk | = |axk |. Den är
√
k
(
)1/k
= |a|1/k |x| .
|bk | = (|bk |)1/k = |axk |
Vi låter k → ∞. För att begripa vilket gränsvärde vi får, logaritmerar vi.
1
· ln |a| + ln |x| → 0 + ln |x| , d.v.s. lim |bk |1/k = |x| .
k→
k
Vi vet att den geometriska serien är konvergent omm |x| < 1.
Serier med teckenalternerande termer
∞
(−1)k+1
har vi tidigare räkna ut. Termerna är ak =
k2
k=1
∞
1
1
och därmed är |ak | = 2 . Nu är ∑ 2 konvergent, Man säger då att
k
k=1 k
Exempel 5.9
Summan
∑
(−1)k+1
k2
∞
(−1)k+1
∑ k2 är absolutkonvergent. Detta är alltså en definition. Man har därmed
k=1
inte sagt att denna summa är konvergent. Däremot kan man bevisa att en abstolukonvergent serie också är konvergent.
∞
Exempel 5.10
Serien
∞
serie är nämligen
∞
(−1)k+1
är inte absolutkonvergent. Motsvarande
k
k=1
∑ ak = ∑
k=1
∑ |ak | den harmoniska serien, som är divergent. Serien är fak-
k=1
tiskt konvergent. Detta följer av nedanstående sats.
(Leibniz kriterium) Om ak ,
k = 1, 2, ... är en följd sådan att
5.2. POTENSSERIER
81
(i) ak och ak+1 har olika tecken och
(ii) |ak | är avtar mot 0, då k → ∞
∞
så är serien
∞
En serie
∑ ak konvergent.
k=1
∑ ak ,
som är konvergent men inte absolutkonvergent, kallas betingat
k=1
konvergent. En sådan serie har "oändlig positiv och negativ massa". För serien
i det sista exemplet menas med detta att
(
)
1 1 1
1 1 1
+ + + ... = ∞ och −
+ + + ... = −∞ .
1 3 5
2 4 6
Genom omordning av termerna kan seriens summa bli vilket tal som helst (inklusive ±∞).
5.2 Potensserier
Definition 5.1
En serie
∞
∑ ak (x − x0 )k
(5.3)
k=0
kallas potensserie.
Kommentarer
• En Tayloserie är en potensserie.
• Vi studerar här främst fallet x0 = 0, d.v.s. serier som motsvarar Maclaurinserier.
Exempel 5.11
Summan
∞
(−1)k−1 xk
= ln(x + 1) för |x| < 1 .
k
k=1
∑
För |x| > 1 konvergerar inte serien. Vi skall använda kvot- och rotkriterierna för
(−1)k−1 xk
att verifiera detta. Vårt ak =
så måste använda |ak | i ställe för ak . För
k
rotkriteriet skriver vi
|ak+1 |
k
1
=
· |x| =
· |x| → |x| =: ρ
|ak |
k+1
1 + 1/k
då k → ∞. Serie när alltså absolutkonvergent för |x| < 1 och divergent för |x| > 1.
För x på "randen", d.v.s. x = ±1, vet vi inte om konvergens föreligger. Vi säger att
konvergensradien är 1.
Av föregående exempel framgår inte klart vad konvergensradie är och hur den
bestäms. Därför ett exempel till.
Exempel 5.12
∞
( x )k
k=0
2
∑k
(a) För vilka x är serien konvergent?
(b) Beräkna seriens summa för de x som summan är konvergent.
KAPITEL 5. TALFÖLJD OCH SERIE
82
Lösning:
( x )k
(a) Vi använder kvotkriteriet. Sätt ak = k
2
.
|ak+1 | (k + 1)|x| 1 + 1/k
|x|
=
=
· |x| →
=: ρ
|ak |
2k
2
2
|x|
då k → ∞. Serien konvergerar alltså absolut om
< 1, d.v.s. om |x| < 2.
2
VI har att konvergensradien är R := 2. Vi vet inte om serien konvergerar för
|x| = 2 men att den divergerar för |x| > 2. Dock med valet x = ±2 gäller att
ak ̸ →0, och alltså divergent.
(b) Beräkna seriens summa för de x som summan är konvergent. Seriens summa
kan beräknas, ty man kan derivera och integrera innanför konvergensradien,
d.v.s. för x, sådana att |x| < 2. Vi fixar till summan så att termerna ser ut
som derivator av potensfunktioner.
∫ x
∞
f (x) = x · ∑ 2−k · k xk−1 ,
k=0
{z
|
G(x) :=
2−k k t k−1 dt =
0
}
=:g(x)
∞
=
2
1
∑ (x/2)k = 1 − x/2 = 2 − x .
k=1
Alltså är
g(x) =
2
så att
(x − 2)2
∞
( x )k
k=0
2
∑k
= x·
2
2x
=
.
2
(x − 2)
(x − 2)2
om x < 2.
Exempel 5.13
Man kan ibland få fram summan av en serie genom att se likheten
med en redan känd series summa. Ex.vis serien
∞
f (x) :=
(−x)k
∑ 2k + 1 för x > 0.
k=0
Beräkna f (1/3).
Lösning: Den liknar serien
∞
(−1)k x2k+1
2k + 1
k=0
∑
som konvergerar mot arctan x för |x| < 1. Vi gör följande omskrivning av den första
serien.
∞
(−1)k · xk
∑ 2k + 1
k=0
För 0 < x < 1 kan vi skriva serien som
√ 2k+1
√
1 ∞
arctan x
k ( x)
√ ∑ (−1) ·
√
=
.
x k=0
2k + 1
x
som är konvergent för 0 < x < 1 enligt tidigare resultat.
vi
√
arctan(1/ 3) π √
√
f (1/3) =
= · 3=
6
1/ 3
Speiclellt för x = 1/3 får
π
√ .
2 3
5.2. POTENSSERIER
83
Kommentarer
• Den ursprungliga serien konvergerar för (åtminstone) |x| < 1.
• Ex.vis är f (0) = 1. Det skall stämma även för
√
arctan x
√
.
x
√
arctan x
√
Vi kan inte direkt sätta in x = 0 i
eftersom vi därmed måste dix
videra med 0. Istället kan vi beräkna gränsvärdet
√
arctan x
√
lim
x→0+
x
√
ex.vis geonom att byta x = t. Vi får
arctant
t − t 3 /3 + t 5 /5 − ... 1/t
= lim
·
= 1.
t→0+
t→0+
t
t
1/t
lim
• För en potensserie med element bk = ak · xk , där ak inte beror på x, kan
man beräkna konvergensradien genom att beräkna ex.vis med rotkriteriet
lim |ak |1/k =: η. Nu är
k→∞
|bk |1/k = |ak |1/k · |x| .
Kriteriet för absolutkonvergens är då η · |x| < 1, så att vi får
|x| <
1
=: R
η
som konvergensradie för potensserien
∞
∑ ak x k
k=1
som alltså är absolutkonvergent för |x| < 1/η och divergent för |x| > 1/η.
• Det är inte alltid som gränvärden av typ lim |bk |1/k =: η existerar. Man kan
k→∞
då ersätta gränsvärdet med lim sup |bk |1/k ("limes superior").
84
KAPITEL 5. TALFÖLJD OCH SERIE
Kapitel 6
Numerisk beräkning av bestämd
integral
Integral är, historiskt sett, äldre än derivata. Ordningen med derivata före integral
är i modern tid emellertid den vanliga. Den hör boken behandlar inte derivata. De
numera använda beteckningarna härrör från matematikern Leibniz, samtida med
Newton. De utvecklade, oberoende av varandra, derivata- och integralbegreppen,
det man ibland kallar integral- och differentialkalkyl. En enkel beskrivning av
integral är beräkning av en ytas area.
6.1 Bakgrund
(Fysik) I ett v − t diagram med konstant hastighet ges tillryggalagd sträcka då a ≤
t ≤ b av den area som begrönsas av grafen och tidsaxeln, samt av linjerna t = a
och t = b (som i figur 6.4). Eftersom detta är en rektangelyta är dess area lätt att
beräkna. Svårare är det dock om grafen ej är en linje. För att lösa sådana problem
har man introducerat integralbegreppet.
6.2 Integral som area
I princip kan vi bara ett fåtal formler för area från geometrin. M.h.a. area för
generaliseras nu begreppet area, till ytan mellan funktionskurva och x− axeln, dör
x ligger i ett givet kompakt intervall (a ≤ x ≤ b).
Denna geometriska tolkning ör den mest grundlöggande för integraler.
6.2.1
Grundidön för berökning av integral
I princip gäller det att berä?kna (skatta) arean mellan funktionskurva (y = f (x)),
x−axel då x ligger i ett intervall [a, b]. Grunden till denna area-ber?kning bygger
på skattning av arean med rektangelytors areor, som n?stan sammanfaller med den
s?kta ytans area (se figur 6.1). Vi ser att om man ritar ett fåtal rektanglar, så får
man en grov skattning av arean men om man har många rektanglar blir skattningen
bättre. Det finns nu i princip två helt olika metoder att beräkna en area.
1. Numerisk beräkning
2. Med primitiv funktion
Exempel 6.1
Antag att man färdas med den konstanta hastigheten 30 km/h tiden 4
h.
Hur långt kommer man då under tiden 1.0 ≤ t ≤ 2.5 h? Detta är givetvis lätt att
85
86
KAPITEL 6. NUMERISK BERÄKNING AV BESTÄMD INTEGRAL
y
x
y
x
Figur 6.1: Approximation med rektangelytors areor
beräkna. Ritar vi grafen y = v(t) := 30 i ett v − t diagram, d.v.s. i ett koordinatsystem (figur 6.4), ser vi att str?ckan kan tolkas som en area. Vi tecknar denna area
som en integral:
∫ 2.5
s=
∫ 2.5
v(t)dt =
1
1
30dt = 30(2.5 − 1) = 45 (km)
Exempel 6.2
Vid konstant hastighetsökning, d.v.s. konstant acceleration a, ex.vis
2
a = 5.0 m/s och med en begynnelsehastighet, d.v.s en hastighet vid starten v0 = 1.0
m/s, blir momentanhastigheten v = v(t) = v0 + at.
För att beräkna tillryggalagd sträcka i tidsintervallet t ∈ [1, 2.5] (s) beräknar vi
motsvarande area under kurvan y = v(t). Även i detta exempel tecknar vi arean
som en integral:
∫ 2.5
s=
∫ 2.5
v(t)dt =
1
1
∫ 2.5
(v0 + at)dt =
(1.0 + 5.0t)dt
1
Arean, d.v.s. integralen blir, m.h.a. (??) sidan ??, 14.625 (m). Konstant acceleration gäller vid fritt fall utan luftmotst?nd.
Så långt räcker våra kunskaper i geometri för att beräkna arean.
Exempel 6.3
Vid fritt fall med luftmotst?nd (h?r ?r begynnelsehastigheten vald till
= 0) kan hastigheten ha utseendet (50 är sluthastighet given i m/s.).
(
)
v = v(t) = 50 1 − e−0.2t m/s
6.2. INTEGRAL SOM AREA
87
y
x
Figur 6.2: En (best?md) integral tolkas som area s?n?r som p? tecken
(km/h)
y
30
t
1.0
2.5
t
(h)
Figur 6.3: Konstant hastighet
(m/s)
y
1.0
t
1.0
2.5
t
(s)
Figur 6.4: Konstant acceleration; v = v0 + at
KAPITEL 6. NUMERISK BERÄKNING AV BESTÄMD INTEGRAL
88
v
m/s
s
s
t
(
)
Figur 6.5: Kurvan v = v(t) = 50 1 − e−0.2t
Kurvan är återgiven i figur 6.5. Att beräkna sträckan geometriskt, d.v.s. som
arean(!) mellan t−axeln och kurvan y = v(t) ex.vis för t ∈ [1, 2.5] (sekunder)
(Markerat med grått i samma figur), som vi gjort innan, är nu i det närmaste
omöjligt. Vi tolkar dock arean som en sträcka och tecknar formellt sträckan som
en integral:
∫ 2.5
∫ 2.5 (
)
s=
v(t)dt =
50 1 − e−0.2t dt
1
1
Vi skall lära oss lösa denna och andra integraler med numeriska metoder.
För att allmänt beräkna arean mellan funktionskurva, x−axel då a ≤ x ≤ b utvecklar
vi nu numeriska metoder.
Exempel 6.4
Betrakta funktionen f (x) = kx + m(≥ 0) (figur 6.6). Genom att
beräkna arean mellan y = f (x), y = 0, d?r a ≤ x ≤ b, rent geometriskt, erhåller
vi det som visar sig vara ett specialfall av ett mer generellt resultat.
Arean A mellan kurvan (linjen) y = kx + m kan vi beräkna m.h.a.
den stora triangelns area As = kb · b/2, den lilla triangelns area Al = ka · a/2 och
rektangelns area Ar = m(b − a).
∫ b
a
k
(kx + m)dx = A = As − Al + Ar = (b2 − a2 ) + m(b − a) =
2
(
) (
)
k 2
k 2
=
b + mb −
a + ma
2
2
(6.1)
k
Vi ser att b2 + mb är funktionsvärdet av funktionen
2
k
k
F(x) := x2 + mx i x = b och a2 + ma ?r funktionsvärdet av sammma funktion i
2
2
x = a. Det sista ledet i (6.1) uttrycker vi med insättningsformeln:
(
) (
) [
]b
k 2
k 2
k 2
b + mb −
a + ma = x + mx
2
2
2
a
Om man kan konsten att derivera så ser man också att
(
)
k 2
′
F (x) = D
x + mx = kx + m = f (x)
2
6.3. NUMERISK INTEGRATION
89
y
y=kx+m
y=m
As
Al
Ar=m(b-a)
a
b
x
Figur 6.6: Kurvan y = kx + m där kx + m > 0.
Sammanfattningsvis kan vi skriva
]b
(
)
[
∫ b
k 2
k 2
k 2
(kx + m)dx = x + mx = b + mb −
a + ma
2
2
2
a
a
(6.2)
6.3 Numerisk integration
Som tidigare påpekats kan man beräkna integraler numeriskt. Vi skall titta pä tre
metoder, där de två första är till för att visa principen f?r numerisk beräkning av
integraler. Den tredje metoden är mycket effektiv (Simpsons formel).
6.3.1
Rektangelmetoden
Den enklaste areaberäkningen fås om man vet sidorna (sidolängderna) på en rektangel. Därför är denna metod, att approximera arean med rektanglar (Se figur),
den enklaste numeriska metoden, Den heter Rektangelmetoden och kallas också
Mittpunktsformeln.
Exempel 6.5
Börja med att rita kurvan y = 1/x = f (x) för 1 ≤ x ≤ 4. Antag att vi
vill beräkna arean mellan kurvan och x−axeln d? a) 1 ≤ 2, b) 1 ≤ 3.
Lösning:
a) Vi delar in intervallet {x : 1 ≤ x ≤ 2} = [1, 2] = [a, b] i ett antal lika långa
delintervall. Låt oss ta två delintervall. Dessa är således [1, 1.5] och [1.5, 2].
Observera att punkten 1.5 fås genom att ta medelvärdet av 1 och 2:
1+2 3
= = 1.5
2
2
I var och en av dessa intervall tar den mittersta punkten, d.v.s. x1 = 1.25 och
x2 = 1.75. I dessa punkterna beräknar vi sedan rektanglarnas höjder som är
f (1.25) respektive f (1.75). Här avses de två rektanglarna som är längst till
vänster i figuren. Den sammanlagda arean av de två rektanglarna är alltså
(
)
1
1
A = 0.5 · ( f (1.25) + f (1.75)) = 0.5 ·
+
≈ 0.69
1.25 1.75
Man inser att om man tar fler delintervall, så får man bättre noggrannhet.
KAPITEL 6. NUMERISK BERÄKNING AV BESTÄMD INTEGRAL
90
b) Vi använder oss av fyra rektanglar och kan alltså använda de två rektanglarnas från a). Arean blir
A = 0.5 · ( f (1.25) + f (1.75) + f (2, 25) + f (2.75)) =
(
1
1
1
1
0.5 ·
+
+
+
1.25 1.75 2.25 2.75
)
≈ 1.09
Om vi antar att vi har en funktion f (x), ett intervall [a, b] och att vi delar in intervallet i två (n = 2) lika långa delintervall, så får vi en formel (ett uttryck) för summan
a+b
(Se exempel 6.5). Vi
av rektanglarnas area: Mittpunkten i intervallet [a, b] är
2
a+b
sätter punkten a = x0 ,
= x1 och b = x2 = xn . Indexet på b är alltså 2 som är
2
lika med antal delintervall. Nu tar vi mittpunkten av dessa intervall:
x0 + x1
x1 + x2
och
2
2
Därefter beräknar vi funktionsvärdena i dessa punkter och multiplicerar med interb−a
vallens gemensamma längd, som är
:
2
[ (
)
(
)]
b−a
x0 + x1
x1 + x2
A=
f
+f
2
2
2
Vi delar in [a, b] i n lika långa delintervall:
a = x0 < x1 < x2 < . . . < xn−1 < xn = b
där
b−a
=h
n
Summan av rektangelareorna för en funktion f (x) på intervallet
[ (
)
(
)
(
)]
x0 + x1
x1 + x2
xn−1 + xn
b−a
f
+f
+...+ f
A=
n
2
2
2
x1 − x0 = x2 − x1 = . . . = xn − xn−1 =
(6.3)
Övningar
6.6
Beräkna följande integraler med Rektangelmetoden, där funktion (integrand)
och antal delintervall n är givna nedan.
∫ 2
a)
x2 dx,
n=2
b)
0
∫ 1
c)
6.7
1
x
e dx,
0
∫ 2
1
x
dx,
n=4
∫ π/2
n=2
d)
sin x dx,
n=2
0
Beräkna den sammanlagda arean av rektanglarna i figuren. Teckna den integral
som du därmed approximerar. (Höjderna beräknas med miniräknare.)
6.3. NUMERISK INTEGRATION
6.3.2
91
Sekantmetoden
Sekantmetoden (eller Trapetsformeln) är i regel effektivare än Rektangelmetoden,
d.v.s. ger en mer noggrann uppskattning av arean. Sekantmetoden går ut på att
dra linjer mellan konsekutiva punkter på kurvan som i x−led ligger på ett konstant
b−a
avstånd h =
. De ytor som erhålls är då parallelltrapets vars area ges av (??)
n
på sidan ??.
Exempel 6.6
1 ≤ x ≤ 3.
Uppskatta arean med Sekantmetoden som i exempel 6.5 b), d.v.s. d?
Lösning: Vi använder oss av fyra delintervall vilket exakt motsvarar bilden. Den
första ytans area är
(1.5 − 1) ·
f (1) + f (1.5)
1 + 1/1.5
= 0.5 ·
2
2
För att rationalisera arbetet sätter vi
a = 1 = x0 , 1.5 = x1 , 2.0 = x2 , 2.5 = x3 , 3.0 = x4 = b
samt h =
b−a
där n = 4. Arean av det första trapetset är med dessa beteckningar
n
A1 = h ·
f (x0 ) + f (x1 )
2
De andra trapetsen har areorna
A2 = h ·
f (x2 ) + f (x3 )
f (x1 ) + f (x2 )
, A3 = h ·
,
2
2
A4 = h ·
f (x3 ) + f (x4 )
2
Den totala arean ä?r alltså A1 + A2 + A3 + A4 , d.v.s.
(
f (x0 ) + f (x1 ) f (x1 ) + f (x2 )
A = h·
+
+
2
2
f (x2 ) + f (x3 ) f (x3 ) + f (x4 )
+
2
2
)
Detta går att förenkla till
A = h · ( f (x0 )/2 + f (x1 ) + f (x2 ) + f (x3 ) + f (x4 )/2)
Alltså blir arean med siffor som ovan
0.5 · (1/2 + 1/1.5 + 1/2.0 + 1/2.5 + (1/3.0)/2) ≈ 1.12
För n delintervall [x0 , x1 ], . . . , [xn−1 , xn ] blir integralen
(
)
∫ b
f (xn )
b − a f (x0 )
+ f (x1 ) + f (x2 ) + . . . + f (xn−1 ) +
f (x)dx ≈
n
2
2
a
dä?r x0 = a och xn = b, samt xk − xk−1 =
b−a
.
n
(6.4)
KAPITEL 6. NUMERISK BERÄKNING AV BESTÄMD INTEGRAL
92
6.3.3
Simpsons formel
Simpsons formel1 är som tidigare påpekats mycket effektiv. Den liknar Sekantmetoden (6.4). Den utnyttjar tre konsekutiva punkter och bildar med dessa en
funktion y = ax2 + bx + c. Längst till vänster utnyttjas (x1 , f (x1 ), (x2 , f (x2 ) och
(x3 , f (x3 ) för att bilda en funktion på formen y = ax2 + bx + c. Därefter utnyttjas
(x2 , f (x2 ), (x3 , f (x3 ) och (x4 , f (x4 )) för att bilda ytterligare en funktion på formen
y = ax2 + bx + c. Speciellt måste antal delintervall vara jämnt. Vi gör inte in i detalj
på hur (6.5) följer utifrån detta.
∫ b
a
f (x)dx ≈
b−a
( f (x0 ) + 4 f (x1 ) + 2 f (x2 ) + . . . + 4 f (xn−1 ) + f (xn ))
3n
(6.5)
Antal delintervall är alltså 2, 4, 6, 8, . . . etc. Vi betecknar därför antalet delintervall
2n (i stället för n). (6.5) går därför att skriva
∫ b
a
f (x)dx ≈
b−a
( f (a) + f (b) + 4( f (x1 ) + f (x3 ) + . . . + f (x2n−1 ))+
6n
2( f (x2 ) + f (x4 ) + . . . + f (x2n−2 ))
(6.6)
1 Thomas
Simpson 1710-1761
6.3. NUMERISK INTEGRATION
93
Exempel 6.7
Vi tillämpar (6.6) på exempel 6.5 b). Alltså är funktionen y = 1/x =
f (x) och intervallet är [1, 3] = [a, b], som delas in i 4 = 2n (fyra) lika långa delinb−a 3−1 1
=
= . Punkterna x0 , . . . , x2n är alltså
tervall. Observera att
6n
12
6
a = 1.0, b = 3.0, x1 = 1.5, x2 = 2.0, x3 = 2.5
Högerledet i (6.6) blir då
1
( f (1.0) + f (3.0) + 4( f (1.5) + f (2.5)) + 2 f (2.0)) = 1.04
6
Kommentarer
• Det finns ett exakt värde på
∫ 3
1
· dx och det är ln 3 ≈ 1.09861. Då är det
x
faktiskt rektangelmetoden som ger det bästa resultatet, tvärtemot det faktum
att Simpsons formel i allmänhet är bäst!
1
• Av de ovan beskrivna metoderna är det i princip Simpsons formel som används i miniräknarna. Exakt hur miniräknaren beräknar en integral numeriskt behöver vi dock inte bekymra oss om. Förhopp- ningsvis har detta
kapitel gett en insikt hur, i stora drag, de numeriska beräkning- arna av integraler utförs.